Re: [algogeeks] Amazon

2011-07-19 Thread ankit sambyal
for the 1st:
Divide the base into 5 segments and join the 4 points on the base to
the vertex opposite to the base.
The 5 triangles formed have equal area because area=1/2 * base * altitude

-- 
You received this message because you are subscribed to the Google Groups 
Algorithm Geeks group.
To post to this group, send email to algogeeks@googlegroups.com.
To unsubscribe from this group, send email to 
algogeeks+unsubscr...@googlegroups.com.
For more options, visit this group at 
http://groups.google.com/group/algogeeks?hl=en.



Re: [algogeeks] Amazon

2011-07-19 Thread ankit sambyal
The above solution I posted was for question 2

-- 
You received this message because you are subscribed to the Google Groups 
Algorithm Geeks group.
To post to this group, send email to algogeeks@googlegroups.com.
To unsubscribe from this group, send email to 
algogeeks+unsubscr...@googlegroups.com.
For more options, visit this group at 
http://groups.google.com/group/algogeeks?hl=en.



Re: [algogeeks] Amazon

2011-07-19 Thread SAMM
FOR question1

it returns no of set bits of N.

On 7/19/11, oppilas . jatka.oppimi...@gmail.com wrote:
 For 3rd, if we only want winner,
 Then 1110 players must lose. So total 1110 games.

 On Tue, Jul 19, 2011 at 10:37 AM, sagar pareek sagarpar...@gmail.comwrote:

 1. Let's say

 S=D=N

 repeat

 D=(floor)D/2;

 S=S-D;

 till D=0

 From the above logic, figure out which algorithm is followed by 'S'

 2. How can you divide a triangle, to 5 equal triangles?

 3. There are  players for a chess tournament, having every game as a
 knock out game (a player will

 be out of the tournament if he/she loses a game), how many games need to
 be
 conducted to determine a
 winner of the tournament.

 --
 Regards
 SAGAR PAREEK
 COMPUTER SCIENCE AND ENGINEERING
 NIT ALLAHABAD

 --
 You received this message because you are subscribed to the Google Groups
 Algorithm Geeks group.
 To post to this group, send email to algogeeks@googlegroups.com.
 To unsubscribe from this group, send email to
 algogeeks+unsubscr...@googlegroups.com.
 For more options, visit this group at
 http://groups.google.com/group/algogeeks?hl=en.


 --
 You received this message because you are subscribed to the Google Groups
 Algorithm Geeks group.
 To post to this group, send email to algogeeks@googlegroups.com.
 To unsubscribe from this group, send email to
 algogeeks+unsubscr...@googlegroups.com.
 For more options, visit this group at
 http://groups.google.com/group/algogeeks?hl=en.




-- 
Somnath Singh

-- 
You received this message because you are subscribed to the Google Groups 
Algorithm Geeks group.
To post to this group, send email to algogeeks@googlegroups.com.
To unsubscribe from this group, send email to 
algogeeks+unsubscr...@googlegroups.com.
For more options, visit this group at 
http://groups.google.com/group/algogeeks?hl=en.



Re: [algogeeks] Amazon

2011-07-19 Thread SAMM
FOR question1

it returns no of set bits of N.

On 7/19/11, SAMM somnath.nit...@gmail.com wrote:
 FOR question1

 it returns no of set bits of N.

 On 7/19/11, oppilas . jatka.oppimi...@gmail.com wrote:
 For 3rd, if we only want winner,
 Then 1110 players must lose. So total 1110 games.

 On Tue, Jul 19, 2011 at 10:37 AM, sagar pareek
 sagarpar...@gmail.comwrote:

 1. Let's say

 S=D=N

 repeat

 D=(floor)D/2;

 S=S-D;

 till D=0

 From the above logic, figure out which algorithm is followed by 'S'

 2. How can you divide a triangle, to 5 equal triangles?

 3. There are  players for a chess tournament, having every game as a
 knock out game (a player will

 be out of the tournament if he/she loses a game), how many games need to
 be
 conducted to determine a
 winner of the tournament.

 --
 Regards
 SAGAR PAREEK
 COMPUTER SCIENCE AND ENGINEERING
 NIT ALLAHABAD

 --
 You received this message because you are subscribed to the Google
 Groups
 Algorithm Geeks group.
 To post to this group, send email to algogeeks@googlegroups.com.
 To unsubscribe from this group, send email to
 algogeeks+unsubscr...@googlegroups.com.
 For more options, visit this group at
 http://groups.google.com/group/algogeeks?hl=en.


 --
 You received this message because you are subscribed to the Google Groups
 Algorithm Geeks group.
 To post to this group, send email to algogeeks@googlegroups.com.
 To unsubscribe from this group, send email to
 algogeeks+unsubscr...@googlegroups.com.
 For more options, visit this group at
 http://groups.google.com/group/algogeeks?hl=en.




 --
 Somnath Singh



-- 
Somnath Singh

-- 
You received this message because you are subscribed to the Google Groups 
Algorithm Geeks group.
To post to this group, send email to algogeeks@googlegroups.com.
To unsubscribe from this group, send email to 
algogeeks+unsubscr...@googlegroups.com.
For more options, visit this group at 
http://groups.google.com/group/algogeeks?hl=en.



Re: [algogeeks] Amazon

2011-07-19 Thread SkRiPt KiDdIe
I think equal is referred as congruent.

-- 
You received this message because you are subscribed to the Google Groups 
Algorithm Geeks group.
To post to this group, send email to algogeeks@googlegroups.com.
To unsubscribe from this group, send email to 
algogeeks+unsubscr...@googlegroups.com.
For more options, visit this group at 
http://groups.google.com/group/algogeeks?hl=en.



Re: [algogeeks] Amazon

2011-07-19 Thread sagar pareek
hey frnds :)
isn't the first one is zero knapsack problem?

On Tue, Jul 19, 2011 at 11:42 AM, SkRiPt KiDdIe anuragmsi...@gmail.comwrote:

 I think equal is referred as congruent.

  --
 You received this message because you are subscribed to the Google Groups
 Algorithm Geeks group.
 To post to this group, send email to algogeeks@googlegroups.com.
 To unsubscribe from this group, send email to
 algogeeks+unsubscr...@googlegroups.com.
 For more options, visit this group at
 http://groups.google.com/group/algogeeks?hl=en.




-- 
**Regards
SAGAR PAREEK
COMPUTER SCIENCE AND ENGINEERING
NIT ALLAHABAD

-- 
You received this message because you are subscribed to the Google Groups 
Algorithm Geeks group.
To post to this group, send email to algogeeks@googlegroups.com.
To unsubscribe from this group, send email to 
algogeeks+unsubscr...@googlegroups.com.
For more options, visit this group at 
http://groups.google.com/group/algogeeks?hl=en.



Re: [algogeeks] Some basic bit fiddling

2011-07-19 Thread sagar pareek
ok

On Tue, Jul 19, 2011 at 11:05 AM, Neeraj Gupta neeraj.gupta...@gmail.comwrote:

 Sagar, when we shift a variable beyond it's width,then output becomes
 dependent on the compiler,so it is undefined behavior.
 In Dev Cpp, I was getting 4 as an output.but there was a warning that left
 shift is greater than the width of the variable.So I checked on ideone
 http://ideone.com/Dwof8


 On Tue, Jul 19, 2011 at 10:29 AM, sagar pareek sagarpar...@gmail.comwrote:

 anybody pls if know give the answer


 On Tue, Jul 19, 2011 at 8:02 AM, Neeraj Gupta 
 neeraj.gupta...@gmail.comwrote:

 UB.
 http://ideone.com/Dwof8


 On Tue, Jul 19, 2011 at 12:26 AM, Ankur Khurana 
 ankur.kkhur...@gmail.com wrote:


 please try this

 cout(134); // gives output 0

 int i=1;
 i=i34;
 couti ; //gives 4 as out put , why ?


 --
 Ankur Khurana
 Computer Science , 4th year
 Netaji Subhas Institute Of Technology
 Delhi.

  --
 You received this message because you are subscribed to the Google
 Groups Algorithm Geeks group.
 To post to this group, send email to algogeeks@googlegroups.com.
 To unsubscribe from this group, send email to
 algogeeks+unsubscr...@googlegroups.com.
 For more options, visit this group at
 http://groups.google.com/group/algogeeks?hl=en.


  --
 You received this message because you are subscribed to the Google Groups
 Algorithm Geeks group.
 To post to this group, send email to algogeeks@googlegroups.com.
 To unsubscribe from this group, send email to
 algogeeks+unsubscr...@googlegroups.com.
 For more options, visit this group at
 http://groups.google.com/group/algogeeks?hl=en.




 --
 **Regards
 SAGAR PAREEK
 COMPUTER SCIENCE AND ENGINEERING
 NIT ALLAHABAD

  --
 You received this message because you are subscribed to the Google Groups
 Algorithm Geeks group.
 To post to this group, send email to algogeeks@googlegroups.com.
 To unsubscribe from this group, send email to
 algogeeks+unsubscr...@googlegroups.com.
 For more options, visit this group at
 http://groups.google.com/group/algogeeks?hl=en.


  --
 You received this message because you are subscribed to the Google Groups
 Algorithm Geeks group.
 To post to this group, send email to algogeeks@googlegroups.com.
 To unsubscribe from this group, send email to
 algogeeks+unsubscr...@googlegroups.com.
 For more options, visit this group at
 http://groups.google.com/group/algogeeks?hl=en.




-- 
**Regards
SAGAR PAREEK
COMPUTER SCIENCE AND ENGINEERING
NIT ALLAHABAD

-- 
You received this message because you are subscribed to the Google Groups 
Algorithm Geeks group.
To post to this group, send email to algogeeks@googlegroups.com.
To unsubscribe from this group, send email to 
algogeeks+unsubscr...@googlegroups.com.
For more options, visit this group at 
http://groups.google.com/group/algogeeks?hl=en.



Re: [algogeeks] Re: Ever growing sorted linked list

2011-07-19 Thread sagar pareek
hey
check my algo...i mentioned all the possible cases   :)

On Tue, Jul 19, 2011 at 11:31 AM, oppilas . jatka.oppimi...@gmail.comwrote:

  Yes we can avoid integer comparison.

 But for jumping  we will need to check whether the next node is null or
 not.
 So, depending upon the jump size, the number of comparison in worst case
 can be very large if our jump size is increasing exponentially. So, why not
 just compare with the next node instead of jumping.



 On Tue, Jul 19, 2011 at 10:47 AM, sunny agrawal 
 sunny816.i...@gmail.comwrote:

 yes Alok u r right that in any case we will traverse k times if k is the
 position if the element that need to searched
 but by jumping we can save the time of avoiding unnecessary comparisions



 On Tue, Jul 19, 2011 at 10:44 AM, Alok Prakash alok251...@gmail.comwrote:


 If i am not wrong, in a linked list to move to any number of position,
 say n, we have to traverse all intermediate nodes of the linked list.

 So, it does not matter if we are moving pointer by 2,4,8,... positions,
 we have to scan all intermediate nodes.

 Is it not a simple searching



 On Tue, Jul 19, 2011 at 9:17 AM, sumit sumitispar...@gmail.com wrote:

 Here is the idea I was thinking of:

 - start from the root: if(root-data  k) move to position 2 in the
 list.
 - in this way every time move the pointer to the position double the
 current position and compare th eelement of node with k( moving here
 is form 1 - 2 , 2-4, 4-8 ,8-16 ..)
 - suppose you found a certain node at position n whose node-data 
 k , then now u only have to search for k between index n/2 to n (i.e.
 if you found 16th element k then search for k between 8th and 16ht
 element)..

 correct me if any flaws.

 On Jul 19, 3:38 am, Dumanshu duman...@gmail.com wrote:
  @Gaurav: Ever Increasing means that you don't know the length of the
  list. So you have to assume some index n, traverse the list upto that
  point and check the results. If not found increment the n to some
  higher value.
  We are basically trying to improve the complexity by taking higher and
  higher jumps for n.
 
  On Jul 19, 2:03 am, Gaurav Popli abeygau...@gmail.com wrote:
 
 
 
 
 
 
 
   yeah...im saying to reach position n at which k is placed we have to
   trverse n positions from head or not...??
   and i didnt understand the use of ever increasing...please elaborate
 on it..
 
   On Tue, Jul 19, 2011 at 2:08 AM, Dumanshu duman...@gmail.com
 wrote:
@Swathi: plz give the TC of ur algo (exponential plus log)
 
On Jul 18, 10:14 pm, Swathi chukka.swa...@gmail.com wrote:
The solution to this problem will be a combination of exponential
 increase
and the binary search..
 
start = 0;
end = 0;
index =0;
middle = 0;
while (1)
{
  if(a[start] == data)
return start;
  if(a[end] == data)
return end;
  if(data  end)
  {
   start = end;
   end = pow(start,2); // here we are consider exponential for
 faster
search.
   // no need to check any boundary conditions as the array is
 infinite
   continue;
  }
  else
  {
// do binary search between start index and end index because
 data is
inbetween a[start] and a[end]
  }
 
}
 
Hope this helps...
 
On Mon, Jul 18, 2011 at 10:32 PM, Gaurav Popli 
 gpgaurav.n...@gmail.comwrote:
 
 i dont understand it..if k is at position an let supposeso
 to
 check at that position dont you have to traverse till that
 position
 ...is thr anything else than the head of list...??or i
 understood
 wrongly...??
 
 On Mon, Jul 18, 2011 at 9:55 PM, sagar pareek 
 sagarpar...@gmail.com
 wrote:
  Update on 2nd line
 
  2.if( ptr2=ptr1-next-next...(5 or 10 times) ) goto
 3.
 else
  make linear search till NULL encounter and exit with the
 solution
 
  On Mon, Jul 18, 2011 at 7:41 PM, sagar pareek 
 sagarpar...@gmail.com
 wrote:
 
  i have one approach :-
 
  first compare root-data  and k
  if k is very much greater than root-data then set
 next=5or10 ur choice
 
  else set next 2or3  ur choice
  take two pointers ptr1 and ptr2
 
  now lets take k is much greater than root-data
  then
  1. set ptr1=root //initially
  2. if( ptr2=ptr1-next-next...(5 or 10 times) ) else
 make linear
  search till NULL encounter
  3. if ptr2-data==k return its position
  4. else if (ptr2-datak) set ptr1=ptr2 goto 2
  5. else traverse the ptr1 upto ptr2, if found return its
 position else
  return fail
 
  if anyone has more efficient solution then pls tell  :)
  On Mon, Jul 18, 2011 at 6:53 PM, Dumanshu 
 duman...@gmail.com wrote:
 
  You have a sorted linked list of integers of some length
 you don't
  know and it keeps on increasing. You are given a number k.
 Print the
  position of the number k.
  Basically, you have to search for number k in the ever
 growing sorted
  list 

[algogeeks] Re: Ever growing sorted linked list

2011-07-19 Thread Dumanshu
Some please give the TC for exponential + logarithmic search method
pointed out by Swathi.

On Jul 19, 11:51 am, Ankur Khurana ankur.kkhur...@gmail.com wrote:
 @sagar: you might have proposed a solution but it ever occured to you that
 (ptr-next)-next  means that you have gone through two diferent nodes. For
 saving one comparison , we are doing lot of un necessary computations which
 , IMHO is not good or feasible . Linked list are not made for searching
 purpose, for that arrays were made.

 On Tue, Jul 19, 2011 at 12:13 PM, sagar pareek sagarpar...@gmail.comwrote:







  hey
  check my algo...i mentioned all the possible cases   :)

  On Tue, Jul 19, 2011 at 11:31 AM, oppilas . 
  jatka.oppimi...@gmail.comwrote:

   Yes we can avoid integer comparison.

  But for jumping  we will need to check whether the next node is null or
  not.
  So, depending upon the jump size, the number of comparison in worst case
  can be very large if our jump size is increasing exponentially. So, why not
  just compare with the next node instead of jumping.

  On Tue, Jul 19, 2011 at 10:47 AM, sunny agrawal 
  sunny816.i...@gmail.comwrote:

  yes Alok u r right that in any case we will traverse k times if k is the
  position if the element that need to searched
  but by jumping we can save the time of avoiding unnecessary comparisions

  On Tue, Jul 19, 2011 at 10:44 AM, Alok Prakash 
  alok251...@gmail.comwrote:

  If i am not wrong, in a linked list to move to any number of position,
  say n, we have to traverse all intermediate nodes of the linked list.

  So, it does not matter if we are moving pointer by 2,4,8,... positions,
  we have to scan all intermediate nodes.

  Is it not a simple searching

  On Tue, Jul 19, 2011 at 9:17 AM, sumit sumitispar...@gmail.com wrote:

  Here is the idea I was thinking of:

  - start from the root: if(root-data  k) move to position 2 in the
  list.
  - in this way every time move the pointer to the position double the
  current position and compare th eelement of node with k( moving here
  is form 1 - 2 , 2-4, 4-8 ,8-16 ..)
  - suppose you found a certain node at position n whose node-data 
  k , then now u only have to search for k between index n/2 to n (i.e.
  if you found 16th element k then search for k between 8th and 16ht
  element)..

  correct me if any flaws.

  On Jul 19, 3:38 am, Dumanshu duman...@gmail.com wrote:
   @Gaurav: Ever Increasing means that you don't know the length of the
   list. So you have to assume some index n, traverse the list upto that
   point and check the results. If not found increment the n to some
   higher value.
   We are basically trying to improve the complexity by taking higher
  and
   higher jumps for n.

   On Jul 19, 2:03 am, Gaurav Popli abeygau...@gmail.com wrote:

yeah...im saying to reach position n at which k is placed we have
  to
trverse n positions from head or not...??
and i didnt understand the use of ever increasing...please
  elaborate on it..

On Tue, Jul 19, 2011 at 2:08 AM, Dumanshu duman...@gmail.com
  wrote:
 @Swathi: plz give the TC of ur algo (exponential plus log)

 On Jul 18, 10:14 pm, Swathi chukka.swa...@gmail.com wrote:
 The solution to this problem will be a combination of
  exponential increase
 and the binary search..

 start = 0;
 end = 0;
 index =0;
 middle = 0;
 while (1)
 {
   if(a[start] == data)
     return start;
   if(a[end] == data)
     return end;
   if(data  end)
   {
    start = end;
    end = pow(start,2); // here we are consider exponential for
  faster
 search.
    // no need to check any boundary conditions as the array is
  infinite
    continue;
   }
   else
   {
     // do binary search between start index and end index
  because data is
 inbetween a[start] and a[end]
   }

 }

 Hope this helps...

 On Mon, Jul 18, 2011 at 10:32 PM, Gaurav Popli 
  gpgaurav.n...@gmail.comwrote:

  i dont understand it..if k is at position an let supposeso
  to
  check at that position dont you have to traverse till that
  position
  ...is thr anything else than the head of list...??or i
  understood
  wrongly...??

  On Mon, Jul 18, 2011 at 9:55 PM, sagar pareek 
  sagarpar...@gmail.com
  wrote:
   Update on 2nd line

   2.    if( ptr2=ptr1-next-next...(5 or 10 times) ) goto
  3.
  else
   make linear search till NULL encounter and exit with the
  solution

   On Mon, Jul 18, 2011 at 7:41 PM, sagar pareek 
  sagarpar...@gmail.com
  wrote:

   i have one approach :-

   first compare root-data  and k
   if k is very much greater than root-data then set
  next=5or10 ur choice

   else set next 2or3  ur choice
   take two pointers ptr1 and ptr2

   now lets take k is much greater than root-data
   then
   1. set ptr1=root //initially
   2. if( ptr2=ptr1-next-next...(5 

[algogeeks] Re: Amazon

2011-07-19 Thread Dumanshu
What is the answer to the first one?

On Jul 19, 10:07 am, sagar pareek sagarpar...@gmail.com wrote:
 1. Let's say

 S=D=N

 repeat

 D=(floor)D/2;

 S=S-D;

 till D=0

 From the above logic, figure out which algorithm is followed by 'S'

 2. How can you divide a triangle, to 5 equal triangles?

 3. There are  players for a chess tournament, having every game as a
 knock out game (a player will

 be out of the tournament if he/she loses a game), how many games need to be
 conducted to determine a
 winner of the tournament.

 --
 **Regards
 SAGAR PAREEK
 COMPUTER SCIENCE AND ENGINEERING
 NIT ALLAHABAD

-- 
You received this message because you are subscribed to the Google Groups 
Algorithm Geeks group.
To post to this group, send email to algogeeks@googlegroups.com.
To unsubscribe from this group, send email to 
algogeeks+unsubscr...@googlegroups.com.
For more options, visit this group at 
http://groups.google.com/group/algogeeks?hl=en.



Re: [algogeeks] Puzzle[Google] Can be Solved programatically as well

2011-07-19 Thread D!leep Gupta
Ans. *German*

-- 
You received this message because you are subscribed to the Google Groups 
Algorithm Geeks group.
To post to this group, send email to algogeeks@googlegroups.com.
To unsubscribe from this group, send email to 
algogeeks+unsubscr...@googlegroups.com.
For more options, visit this group at 
http://groups.google.com/group/algogeeks?hl=en.



[algogeeks] Re: MICROSOFT

2011-07-19 Thread Dumanshu
@Surender: sorry, i had missed a case. We need a 3 way comparison.
heres the correct version

btree* max_tree(btree *root)
{
if(root == NULL)
return root;
btree * current = root;
while(current-right != NULL)
{
current = current-right;
}
return current;
}

btree * min_tree(btree *root)
{
if(root == NULL)
return root;
btree * current = root;
while(current-left != NULL)
{
current = current-left;
}
return current;
}

void binarytreetobst(btree *root)
{
//base-case tree is empty
if(root == NULL)
return;
else if(root-left == NULL  root-right == NULL) //base-case
tree
of size 1
return;
else
{
binarytreetobst(root-left);
binarytreetobst(root-right);
btree* max = max_tree(root-left);
if(max  max-data  root-data)
{
int temp = root-data;
root-data = max-data;
max-data = temp;
binarytreetobst(root-left);
}
btree* min = min_tree(root-right);
if(min  min-data   root-data)
{
int temp = root-data;
root-data = min-data;
min-data = temp;
binarytreetobst(root-right);
max = max_tree(root-left);
if(max  max-data  root-data)
{
 int temp = root-data;
 root-data = max-data;
 max-data = temp;
 binarytreetobst(root-left);
}
}
}
}

On Jul 19, 8:38 am, surender sanke surend...@gmail.com wrote:
 @Damanshu for
         1
       /   \
      2     3
     /  \
    4   5
   /  \
 6    7

 im ending up at some non BST

 surender







 On Tue, Jul 19, 2011 at 4:06 AM, Dumanshu duman...@gmail.com wrote:
  @Gaurav: The best solution would be to manipulate the given BTree in
  place and get the BST. We don't need a separate tree but convert the
  existing one using the same space occupied by nodes of BT already in
  BST.

  On Jul 19, 2:06 am, Gaurav Popli gpgaurav.n...@gmail.com wrote:
   cant we just dotraverse using recursion and instead of printing it
   just pass to function which is making BST??
   and is this right as someone above said first sort it and then make
  BST...
   dont we want that root of both Tree to be same or something like
  that...??

   On Tue, Jul 19, 2011 at 2:17 AM, Dumanshu duman...@gmail.com wrote:
@Balaji: for third question, were u asked to write the code?

On Jul 18, 10:04 pm, Balaji S balaji.ceg...@gmail.com wrote:
wats the mistake..

--
You received this message because you are subscribed to the Google
  Groups Algorithm Geeks group.
To post to this group, send email to algogeeks@googlegroups.com.
To unsubscribe from this group, send email to
  algogeeks+unsubscr...@googlegroups.com.
For more options, visit this group athttp://
  groups.google.com/group/algogeeks?hl=en.- Hide quoted text -

   - Show quoted text -

  --
  You received this message because you are subscribed to the Google Groups
  Algorithm Geeks group.
  To post to this group, send email to algogeeks@googlegroups.com.
  To unsubscribe from this group, send email to
  algogeeks+unsubscr...@googlegroups.com.
  For more options, visit this group at
 http://groups.google.com/group/algogeeks?hl=en.

-- 
You received this message because you are subscribed to the Google Groups 
Algorithm Geeks group.
To post to this group, send email to algogeeks@googlegroups.com.
To unsubscribe from this group, send email to 
algogeeks+unsubscr...@googlegroups.com.
For more options, visit this group at 
http://groups.google.com/group/algogeeks?hl=en.



Re: [algogeeks] Puzzle[Google] Can be Solved programatically as well

2011-07-19 Thread archita monga
yaa..

On Tue, Jul 19, 2011 at 1:35 PM, alagammai narayanan
alagamma...@gmail.comwrote:

 Yep.. Its German.. Were you guys able to come up with the 5 * 5 matrix...
 As in who lives in which house?What does he drink,smoke etc..


 On Tue, Jul 19, 2011 at 1:31 PM, archita monga kool.arc...@gmail.comwrote:

 German!


 On Tue, Jul 19, 2011 at 1:22 PM, D!leep Gupta 
 dileep.smil...@gmail.comwrote:

 Ans. *German*

 --
 You received this message because you are subscribed to the Google Groups
 Algorithm Geeks group.
 To post to this group, send email to algogeeks@googlegroups.com.
 To unsubscribe from this group, send email to
 algogeeks+unsubscr...@googlegroups.com.
 For more options, visit this group at
 http://groups.google.com/group/algogeeks?hl=en.




 --
 Archita Monga

  --
 You received this message because you are subscribed to the Google Groups
 Algorithm Geeks group.
 To post to this group, send email to algogeeks@googlegroups.com.
 To unsubscribe from this group, send email to
 algogeeks+unsubscr...@googlegroups.com.
 For more options, visit this group at
 http://groups.google.com/group/algogeeks?hl=en.


  --
 You received this message because you are subscribed to the Google Groups
 Algorithm Geeks group.
 To post to this group, send email to algogeeks@googlegroups.com.
 To unsubscribe from this group, send email to
 algogeeks+unsubscr...@googlegroups.com.
 For more options, visit this group at
 http://groups.google.com/group/algogeeks?hl=en.




-- 
Archita Monga

-- 
You received this message because you are subscribed to the Google Groups 
Algorithm Geeks group.
To post to this group, send email to algogeeks@googlegroups.com.
To unsubscribe from this group, send email to 
algogeeks+unsubscr...@googlegroups.com.
For more options, visit this group at 
http://groups.google.com/group/algogeeks?hl=en.



Re: [algogeeks] Re: Amazon

2011-07-19 Thread Nitish Garg
The algo gives the number of set bits in the number as pointed out by SAMMM 
above.

-- 
You received this message because you are subscribed to the Google Groups 
Algorithm Geeks group.
To view this discussion on the web visit 
https://groups.google.com/d/msg/algogeeks/-/lyzigAph1iYJ.
To post to this group, send email to algogeeks@googlegroups.com.
To unsubscribe from this group, send email to 
algogeeks+unsubscr...@googlegroups.com.
For more options, visit this group at 
http://groups.google.com/group/algogeeks?hl=en.



[algogeeks] calculating permutations

2011-07-19 Thread Z
HI,

Is there a method to calculate something like 23!/(7!*10!) i.e.
permutations with repeated elements in such a way that we need not
calculate all the factorials involved? The problem in calculating all
the factorials is in storing them. As 23!  will make even long long in
C++ to overflow, but the required answer won't.

Thanks

-- 
You received this message because you are subscribed to the Google Groups 
Algorithm Geeks group.
To post to this group, send email to algogeeks@googlegroups.com.
To unsubscribe from this group, send email to 
algogeeks+unsubscr...@googlegroups.com.
For more options, visit this group at 
http://groups.google.com/group/algogeeks?hl=en.



Re: [algogeeks] Re: Amazon

2011-07-19 Thread sagar pareek
Ok then
What about  ques 2 ?

On Tue, Jul 19, 2011 at 1:40 PM, Nitish Garg nitishgarg1...@gmail.comwrote:

 The algo gives the number of set bits in the number as pointed out by SAMMM
 above.

 --
 You received this message because you are subscribed to the Google Groups
 Algorithm Geeks group.
 To view this discussion on the web visit
 https://groups.google.com/d/msg/algogeeks/-/lyzigAph1iYJ.

 To post to this group, send email to algogeeks@googlegroups.com.
 To unsubscribe from this group, send email to
 algogeeks+unsubscr...@googlegroups.com.
 For more options, visit this group at
 http://groups.google.com/group/algogeeks?hl=en.




-- 
**Regards
SAGAR PAREEK
COMPUTER SCIENCE AND ENGINEERING
NIT ALLAHABAD

-- 
You received this message because you are subscribed to the Google Groups 
Algorithm Geeks group.
To post to this group, send email to algogeeks@googlegroups.com.
To unsubscribe from this group, send email to 
algogeeks+unsubscr...@googlegroups.com.
For more options, visit this group at 
http://groups.google.com/group/algogeeks?hl=en.



[algogeeks] Re: is it possible to detect the first repeating number in a 2-D array (n X n) in O(n) time ?

2011-07-19 Thread ((** VICKY **))
doing xor of all elements in array[][] should work. you start from a[0]
[0] to a[0][n] then a[1][0] .. when the xor value bcomz 0 then the
corresponding value in arr[i][j] is the first repeated element in the
array. though this code will have two loops and seem as O(n2) it will
terminate once it finds the first repeated no. hope my algo works.
comments plz

On Jul 19, 2:14 am, Dumanshu duman...@gmail.com wrote:
 You have to use parallel computing to find the first repeating number
 in O(n) time if theres nothing special about the 2-D array
 Use n +1 processes, n processes to scan each row and 1 process to scan
 the rows last and next rows first element to check for repetition.
 Each process uses hash table to find the first non repeating number.
 When we have the results from all the processes, do O(n) scanning, and
 output the result for minimum row which wud be the first repetition.

 On Jul 18, 10:42 pm, snehi jain snehijai...@gmail.com wrote:







  this question was asked in an interview.

  Regards
  Snehi

-- 
You received this message because you are subscribed to the Google Groups 
Algorithm Geeks group.
To post to this group, send email to algogeeks@googlegroups.com.
To unsubscribe from this group, send email to 
algogeeks+unsubscr...@googlegroups.com.
For more options, visit this group at 
http://groups.google.com/group/algogeeks?hl=en.



[algogeeks] Re: calculating permutations

2011-07-19 Thread SAMMM
For this you can use pascal triangle . It will give the permutation
value of the the series .
   1
   1   2   1
 1   33   1
   1   4  6 4  1
 1  5 10   10   5  1
1  6 15  20   151


This can easily prepreocessed using dynamic method way .


a[0][i]=1   1=i=n
a[i][0]=1   1=i=n
a[i][j] = a[i-1][j-1]+a[i][j-1]


try this out 

-- 
You received this message because you are subscribed to the Google Groups 
Algorithm Geeks group.
To post to this group, send email to algogeeks@googlegroups.com.
To unsubscribe from this group, send email to 
algogeeks+unsubscr...@googlegroups.com.
For more options, visit this group at 
http://groups.google.com/group/algogeeks?hl=en.



[algogeeks] EXPLAIN THE OUTPUTS

2011-07-19 Thread sameer.mut...@gmail.com
#includestdio.h
int * fun(int a1,int b)
{
int a[2];
a[0]=a1;
a[1]=b;
//int c=5;
printf(%x\n,a[0]);
return a;
}
int main()
{
int *r=fun(3,5);
printf(%d\n,r[0]);
printf(%d\n,r[0]);
}

-- 
You received this message because you are subscribed to the Google Groups 
Algorithm Geeks group.
To post to this group, send email to algogeeks@googlegroups.com.
To unsubscribe from this group, send email to 
algogeeks+unsubscr...@googlegroups.com.
For more options, visit this group at 
http://groups.google.com/group/algogeeks?hl=en.



[algogeeks] INFINITY

2011-07-19 Thread SAMMM
Print the symbol ∞  (INFINITY) through code . Unicode ..

-- 
You received this message because you are subscribed to the Google Groups 
Algorithm Geeks group.
To post to this group, send email to algogeeks@googlegroups.com.
To unsubscribe from this group, send email to 
algogeeks+unsubscr...@googlegroups.com.
For more options, visit this group at 
http://groups.google.com/group/algogeeks?hl=en.



Re: [algogeeks] Memory Allocation

2011-07-19 Thread Piyush Sinha
char a[] or any array refers to a block of memory (not a single memory
location or variable). Analogy to this can be seen in the following fact :-
The memory addresses of an array can't be changed, whereas the content of
the pointer variables, such as the base memory address of the array it
refers to or simply any variable can be changed.

Hence an undefined result persists..if we really want to return an array
from a function we use the following syntax..

*char a[] = Hello;
char *b = (char *)malloc(strlen(a)+1);
strcpy(b,a);
return b;*

here we are returning the base address of the character array, unlike as wat
you were doing previously(previously you were trying to return a block of
memory)

Hope it is clear now... :)




-- 
*Piyush Sinha*
*IIIT, Allahabad*
*+91-7483122727*
* https://www.facebook.com/profile.php?id=10655377926 NEVER SAY
NEVER
*

-- 
You received this message because you are subscribed to the Google Groups 
Algorithm Geeks group.
To post to this group, send email to algogeeks@googlegroups.com.
To unsubscribe from this group, send email to 
algogeeks+unsubscr...@googlegroups.com.
For more options, visit this group at 
http://groups.google.com/group/algogeeks?hl=en.



Re: [algogeeks] INFINITY

2011-07-19 Thread Piyush Sinha
*printf(%c\n,236);*

On Tue, Jul 19, 2011 at 2:45 PM, SAMMM somnath.nit...@gmail.com wrote:

 Print the symbol ∞  (INFINITY) through code . Unicode ..

 --
 You received this message because you are subscribed to the Google Groups
 Algorithm Geeks group.
 To post to this group, send email to algogeeks@googlegroups.com.
 To unsubscribe from this group, send email to
 algogeeks+unsubscr...@googlegroups.com.
 For more options, visit this group at
 http://groups.google.com/group/algogeeks?hl=en.




-- 
*Piyush Sinha*
*IIIT, Allahabad*
*+91-7483122727*
* https://www.facebook.com/profile.php?id=10655377926 NEVER SAY
NEVER
*

-- 
You received this message because you are subscribed to the Google Groups 
Algorithm Geeks group.
To post to this group, send email to algogeeks@googlegroups.com.
To unsubscribe from this group, send email to 
algogeeks+unsubscr...@googlegroups.com.
For more options, visit this group at 
http://groups.google.com/group/algogeeks?hl=en.



Re: [algogeeks] Re: INFINITY

2011-07-19 Thread Piyush Sinha
In Dev C it does

On Tue, Jul 19, 2011 at 3:08 PM, SAMMM somnath.nit...@gmail.com wrote:

 It doesn't display the infinity symbol.

 On Jul 19, 2:24 pm, Piyush Sinha ecstasy.piy...@gmail.com wrote:
  *printf(%c\n,236);*
 
  On Tue, Jul 19, 2011 at 2:45 PM, SAMMM somnath.nit...@gmail.com wrote:
   Print the symbol ∞  (INFINITY) through code . Unicode ..
 
   --
   You received this message because you are subscribed to the Google
 Groups
   Algorithm Geeks group.
   To post to this group, send email to algogeeks@googlegroups.com.
   To unsubscribe from this group, send email to
   algogeeks+unsubscr...@googlegroups.com.
   For more options, visit this group at
  http://groups.google.com/group/algogeeks?hl=en.
 
  --
  *Piyush Sinha*
  *IIIT, Allahabad*
  *+91-7483122727*
  * https://www.facebook.com/profile.php?id=10655377926 NEVER SAY
  NEVER
  *

 --
 You received this message because you are subscribed to the Google Groups
 Algorithm Geeks group.
 To post to this group, send email to algogeeks@googlegroups.com.
 To unsubscribe from this group, send email to
 algogeeks+unsubscr...@googlegroups.com.
 For more options, visit this group at
 http://groups.google.com/group/algogeeks?hl=en.




-- 
*Piyush Sinha*
*IIIT, Allahabad*
*+91-7483122727*
* https://www.facebook.com/profile.php?id=10655377926 NEVER SAY
NEVER
*

-- 
You received this message because you are subscribed to the Google Groups 
Algorithm Geeks group.
To post to this group, send email to algogeeks@googlegroups.com.
To unsubscribe from this group, send email to 
algogeeks+unsubscr...@googlegroups.com.
For more options, visit this group at 
http://groups.google.com/group/algogeeks?hl=en.



Re: [algogeeks] Re: INFINITY

2011-07-19 Thread sagar pareek
http://chexed.com/ComputerTips/asciicodes.php

On Tue, Jul 19, 2011 at 3:10 PM, Piyush Sinha ecstasy.piy...@gmail.comwrote:

 In Dev C it does


 On Tue, Jul 19, 2011 at 3:08 PM, SAMMM somnath.nit...@gmail.com wrote:

 It doesn't display the infinity symbol.

 On Jul 19, 2:24 pm, Piyush Sinha ecstasy.piy...@gmail.com wrote:
  *printf(%c\n,236);*
 
  On Tue, Jul 19, 2011 at 2:45 PM, SAMMM somnath.nit...@gmail.com
 wrote:
   Print the symbol ∞  (INFINITY) through code . Unicode ..
 
   --
   You received this message because you are subscribed to the Google
 Groups
   Algorithm Geeks group.
   To post to this group, send email to algogeeks@googlegroups.com.
   To unsubscribe from this group, send email to
   algogeeks+unsubscr...@googlegroups.com.
   For more options, visit this group at
  http://groups.google.com/group/algogeeks?hl=en.
 
  --
  *Piyush Sinha*
  *IIIT, Allahabad*
  *+91-7483122727*
  * https://www.facebook.com/profile.php?id=10655377926 NEVER SAY
  NEVER
  *

 --
 You received this message because you are subscribed to the Google Groups
 Algorithm Geeks group.
 To post to this group, send email to algogeeks@googlegroups.com.
 To unsubscribe from this group, send email to
 algogeeks+unsubscr...@googlegroups.com.
 For more options, visit this group at
 http://groups.google.com/group/algogeeks?hl=en.




 --
 *Piyush Sinha*
 *IIIT, Allahabad*
 *+91-7483122727*
 * https://www.facebook.com/profile.php?id=10655377926 NEVER SAY
 NEVER
 *

  --
 You received this message because you are subscribed to the Google Groups
 Algorithm Geeks group.
 To post to this group, send email to algogeeks@googlegroups.com.
 To unsubscribe from this group, send email to
 algogeeks+unsubscr...@googlegroups.com.
 For more options, visit this group at
 http://groups.google.com/group/algogeeks?hl=en.




-- 
**Regards
SAGAR PAREEK
COMPUTER SCIENCE AND ENGINEERING
NIT ALLAHABAD

-- 
You received this message because you are subscribed to the Google Groups 
Algorithm Geeks group.
To post to this group, send email to algogeeks@googlegroups.com.
To unsubscribe from this group, send email to 
algogeeks+unsubscr...@googlegroups.com.
For more options, visit this group at 
http://groups.google.com/group/algogeeks?hl=en.



[algogeeks] Re: amazon

2011-07-19 Thread SAMMM
O(n) is possible . Will it serve the purpose or need less than that ???

-- 
You received this message because you are subscribed to the Google Groups 
Algorithm Geeks group.
To post to this group, send email to algogeeks@googlegroups.com.
To unsubscribe from this group, send email to 
algogeeks+unsubscr...@googlegroups.com.
For more options, visit this group at 
http://groups.google.com/group/algogeeks?hl=en.



Re: [algogeeks] Re: amazon

2011-07-19 Thread shilpa gupta
give your algo.

On Tue, Jul 19, 2011 at 3:26 PM, SAMMM somnath.nit...@gmail.com wrote:

 O(n) is possible . Will it serve the purpose or need less than that ???

 --
 You received this message because you are subscribed to the Google Groups
 Algorithm Geeks group.
 To post to this group, send email to algogeeks@googlegroups.com.
 To unsubscribe from this group, send email to
 algogeeks+unsubscr...@googlegroups.com.
 For more options, visit this group at
 http://groups.google.com/group/algogeeks?hl=en.




-- 
Shilpa Gupta

B.Tech. 3rd year
Computer Science and Engineering
MNNIT Allahabad

-- 
You received this message because you are subscribed to the Google Groups 
Algorithm Geeks group.
To post to this group, send email to algogeeks@googlegroups.com.
To unsubscribe from this group, send email to 
algogeeks+unsubscr...@googlegroups.com.
For more options, visit this group at 
http://groups.google.com/group/algogeeks?hl=en.



Re: [algogeeks] Re: amazon

2011-07-19 Thread archita monga
Take 2 node pointers.Move one at the speed of twice the
other(first:node-next,second:node-next-next) when the first pointer
reaches the end of the list,the second will give you the middle node.

On Tue, Jul 19, 2011 at 3:27 PM, shilpa gupta shilpagupta...@gmail.comwrote:

 give your algo.


 On Tue, Jul 19, 2011 at 3:26 PM, SAMMM somnath.nit...@gmail.com wrote:

 O(n) is possible . Will it serve the purpose or need less than that ???

 --
 You received this message because you are subscribed to the Google Groups
 Algorithm Geeks group.
 To post to this group, send email to algogeeks@googlegroups.com.
 To unsubscribe from this group, send email to
 algogeeks+unsubscr...@googlegroups.com.
 For more options, visit this group at
 http://groups.google.com/group/algogeeks?hl=en.




 --
 Shilpa Gupta

 B.Tech. 3rd year
 Computer Science and Engineering
 MNNIT Allahabad


  --
 You received this message because you are subscribed to the Google Groups
 Algorithm Geeks group.
 To post to this group, send email to algogeeks@googlegroups.com.
 To unsubscribe from this group, send email to
 algogeeks+unsubscr...@googlegroups.com.
 For more options, visit this group at
 http://groups.google.com/group/algogeeks?hl=en.




-- 
Archita Monga

-- 
You received this message because you are subscribed to the Google Groups 
Algorithm Geeks group.
To post to this group, send email to algogeeks@googlegroups.com.
To unsubscribe from this group, send email to 
algogeeks+unsubscr...@googlegroups.com.
For more options, visit this group at 
http://groups.google.com/group/algogeeks?hl=en.



Re: [algogeeks] Re: is it possible to detect the first repeating number in a 2-D array (n X n) in O(n) time ?

2011-07-19 Thread Bhanu Kishore
@Venkat. That algorithm doesnt work actually.Try for 9,8,1. At 1 , it
becomes 0.

-- 
You received this message because you are subscribed to the Google Groups 
Algorithm Geeks group.
To post to this group, send email to algogeeks@googlegroups.com.
To unsubscribe from this group, send email to 
algogeeks+unsubscr...@googlegroups.com.
For more options, visit this group at 
http://groups.google.com/group/algogeeks?hl=en.



Re: [algogeeks] Re: amazon

2011-07-19 Thread Manish Pathak
exactly archita.

On Tue, Jul 19, 2011 at 3:33 PM, archita monga kool.arc...@gmail.comwrote:

 Take 2 node pointers.Move one at the speed of twice the
 other(first:node-next,second:node-next-next) when the first pointer
 reaches the end of the list,the second will give you the middle node.


 On Tue, Jul 19, 2011 at 3:27 PM, shilpa gupta shilpagupta...@gmail.comwrote:

 give your algo.


 On Tue, Jul 19, 2011 at 3:26 PM, SAMMM somnath.nit...@gmail.com wrote:

 O(n) is possible . Will it serve the purpose or need less than that ???

 --
 You received this message because you are subscribed to the Google Groups
 Algorithm Geeks group.
 To post to this group, send email to algogeeks@googlegroups.com.
 To unsubscribe from this group, send email to
 algogeeks+unsubscr...@googlegroups.com.
 For more options, visit this group at
 http://groups.google.com/group/algogeeks?hl=en.




 --
 Shilpa Gupta

 B.Tech. 3rd year
 Computer Science and Engineering
 MNNIT Allahabad


  --
 You received this message because you are subscribed to the Google Groups
 Algorithm Geeks group.
 To post to this group, send email to algogeeks@googlegroups.com.
 To unsubscribe from this group, send email to
 algogeeks+unsubscr...@googlegroups.com.
 For more options, visit this group at
 http://groups.google.com/group/algogeeks?hl=en.




 --
 Archita Monga

 --
 You received this message because you are subscribed to the Google Groups
 Algorithm Geeks group.
 To post to this group, send email to algogeeks@googlegroups.com.
 To unsubscribe from this group, send email to
 algogeeks+unsubscr...@googlegroups.com.
 For more options, visit this group at
 http://groups.google.com/group/algogeeks?hl=en.




-- 

Thanks and Regards,

Manish Pathak **
TimesJobs.com
manish.pathak*@*timesgroup.com
Mo.  9015687266

-- 
You received this message because you are subscribed to the Google Groups 
Algorithm Geeks group.
To post to this group, send email to algogeeks@googlegroups.com.
To unsubscribe from this group, send email to 
algogeeks+unsubscr...@googlegroups.com.
For more options, visit this group at 
http://groups.google.com/group/algogeeks?hl=en.



Re: [algogeeks] Re: amazon

2011-07-19 Thread Piyush Sinha
*node *middle(node *head)
{
 node *mid;

 mid = head;
 for(int i = 2;head!=NULL;head=head-next,i++)
 {
 (i%2==1) mid = mid-next;
 }
 return mid;
}*

To find 1/3rd of a list, change (i%2==1) to (i%3==1)i.e. nth node can be
found (i%n==1) (make sure n=no. of nodes)

Now to find 3/4th of a node, we can do following

initial call *threefourth(head,3,4);*

*node *threefourth(node *head,int m,int n)
{
 node *mid,*p;
 p = head-next;
 mid = head;
 while(m)
 {
 for(int i = 2;p!=NULL;p=p-next,i++)
 {
 (i%n==1) mid = mid-next;
 }
 m--;
 if(m==0) return mid;
 else
 mid = mid-next;
 p = head-next;

 }
}*



-- 
*Piyush Sinha*
*IIIT, Allahabad*
*+91-7483122727*
* https://www.facebook.com/profile.php?id=10655377926 NEVER SAY
NEVER
*

-- 
You received this message because you are subscribed to the Google Groups 
Algorithm Geeks group.
To post to this group, send email to algogeeks@googlegroups.com.
To unsubscribe from this group, send email to 
algogeeks+unsubscr...@googlegroups.com.
For more options, visit this group at 
http://groups.google.com/group/algogeeks?hl=en.



Re: [algogeeks] Re: amazon

2011-07-19 Thread Piyush Sinha
Ignore my previous post for find the 3/4th...its actually traversing the
list thriceso it better you traverse the list once and find the number
of nodes in it...then then traverse 3/4th times the number of nodes...

But the algo given by me is efficient if we are required to compute 1/nth of
a node...


-- 
*Piyush Sinha*
*IIIT, Allahabad*
*+91-7483122727*
* https://www.facebook.com/profile.php?id=10655377926 NEVER SAY
NEVER
*

-- 
You received this message because you are subscribed to the Google Groups 
Algorithm Geeks group.
To post to this group, send email to algogeeks@googlegroups.com.
To unsubscribe from this group, send email to 
algogeeks+unsubscr...@googlegroups.com.
For more options, visit this group at 
http://groups.google.com/group/algogeeks?hl=en.



[algogeeks] Re: amazon

2011-07-19 Thread SAMMM
Yaa this will work , you need to handle the case for odd number of
nodes .
For even number of node it will serve the purpose .

Yaa for the second part also you can use the ratio concept

fast pointer : slow pointer = 4:3

slow = ptr-next-next-next
fast = ptr-next-next-next-next


Here also we need to handle the case if the number of node is not a
multiple of 4 .

-- 
You received this message because you are subscribed to the Google Groups 
Algorithm Geeks group.
To post to this group, send email to algogeeks@googlegroups.com.
To unsubscribe from this group, send email to 
algogeeks+unsubscr...@googlegroups.com.
For more options, visit this group at 
http://groups.google.com/group/algogeeks?hl=en.



Re: [algogeeks] Solve it

2011-07-19 Thread Piyush Sinha
I hope it can be solved using DP...check my algo below and give any counter
case if you get it...

1. Make an array S equal to the length of the given array where
S[0] = a[0] and S[1] = max(a[0],a[1])

2. for i:2 to n-1
 S[i] = max(S[i-2]+a[i], S[i-1])

3. return S[n-1]

Hope the above algo works...

On Tue, Jul 19, 2011 at 2:59 PM, sagar pareek sagarpar...@gmail.com wrote:

 Given an array all of whose elements are positive numbesr, find the maximum
 sum of a subsequence with the constraint that no 2 numbers in the sequence
 should be adjecent in the array.

 eg:-
 3 2 7 10 should return (sum of 3 and 10)
 3 2 5 10 7 should returnn 15 (sum of 3,5,7)

 --
 **Regards
 SAGAR PAREEK
 COMPUTER SCIENCE AND ENGINEERING
 NIT ALLAHABAD

  --
 You received this message because you are subscribed to the Google Groups
 Algorithm Geeks group.
 To post to this group, send email to algogeeks@googlegroups.com.
 To unsubscribe from this group, send email to
 algogeeks+unsubscr...@googlegroups.com.
 For more options, visit this group at
 http://groups.google.com/group/algogeeks?hl=en.




-- 
*Piyush Sinha*
*IIIT, Allahabad*
*+91-7483122727*
* https://www.facebook.com/profile.php?id=10655377926 NEVER SAY
NEVER
*

-- 
You received this message because you are subscribed to the Google Groups 
Algorithm Geeks group.
To post to this group, send email to algogeeks@googlegroups.com.
To unsubscribe from this group, send email to 
algogeeks+unsubscr...@googlegroups.com.
For more options, visit this group at 
http://groups.google.com/group/algogeeks?hl=en.



Re: [algogeeks] Re: amazon

2011-07-19 Thread surender sanke
take two ptrs ptr1 and ptr2 pointing to head
move ptr1 until 1/4th of size of list.
move ptr1 and ptr2 until ptr1=null
ptr2 is pointing at 3/4th

surender

On Tue, Jul 19, 2011 at 3:42 PM, SAMMM somnath.nit...@gmail.com wrote:

 Yaa this will work , you need to handle the case for odd number of
 nodes .
 For even number of node it will serve the purpose .

 Yaa for the second part also you can use the ratio concept

 fast pointer : slow pointer = 4:3

 slow = ptr-next-next-next
 fast = ptr-next-next-next-next


 Here also we need to handle the case if the number of node is not a
 multiple of 4 .

 --
 You received this message because you are subscribed to the Google Groups
 Algorithm Geeks group.
 To post to this group, send email to algogeeks@googlegroups.com.
 To unsubscribe from this group, send email to
 algogeeks+unsubscr...@googlegroups.com.
 For more options, visit this group at
 http://groups.google.com/group/algogeeks?hl=en.



-- 
You received this message because you are subscribed to the Google Groups 
Algorithm Geeks group.
To post to this group, send email to algogeeks@googlegroups.com.
To unsubscribe from this group, send email to 
algogeeks+unsubscr...@googlegroups.com.
For more options, visit this group at 
http://groups.google.com/group/algogeeks?hl=en.



Re: [algogeeks] Re: INFINITY

2011-07-19 Thread sagar pareek
yeah in my ubuntu too its not printing  :)

On Tue, Jul 19, 2011 at 3:20 PM, SAMMM somnath.nit...@gmail.com wrote:

 But if we use gcc or g++ . In that case it doesn't print it .. Wht abt
 tht ...


 On Jul 19, 2:40 pm, Piyush Sinha ecstasy.piy...@gmail.com wrote:
  In Dev C it does
 
 
 
 
 
  On Tue, Jul 19, 2011 at 3:08 PM, SAMMM somnath.nit...@gmail.com wrote:
   It doesn't display the infinity symbol.
 
   On Jul 19, 2:24 pm, Piyush Sinha ecstasy.piy...@gmail.com wrote:
*printf(%c\n,236);*
 
On Tue, Jul 19, 2011 at 2:45 PM, SAMMM somnath.nit...@gmail.com
 wrote:
 Print the symbol ∞  (INFINITY) through code . Unicode ..
 
 --
 You received this message because you are subscribed to the Google
   Groups
 Algorithm Geeks group.
 To post to this group, send email to algogeeks@googlegroups.com.
 To unsubscribe from this group, send email to
 algogeeks+unsubscr...@googlegroups.com.
 For more options, visit this group at
http://groups.google.com/group/algogeeks?hl=en.
 
--
*Piyush Sinha*
*IIIT, Allahabad*
*+91-7483122727*
* https://www.facebook.com/profile.php?id=10655377926 NEVER
 SAY
NEVER
*
 
   --
   You received this message because you are subscribed to the Google
 Groups
   Algorithm Geeks group.
   To post to this group, send email to algogeeks@googlegroups.com.
   To unsubscribe from this group, send email to
   algogeeks+unsubscr...@googlegroups.com.
   For more options, visit this group at
  http://groups.google.com/group/algogeeks?hl=en.
 
  --
  *Piyush Sinha*
  *IIIT, Allahabad*
  *+91-7483122727*
  * https://www.facebook.com/profile.php?id=10655377926 NEVER SAY
  NEVER
  *- Hide quoted text -
 
  - Show quoted text -

 --
 You received this message because you are subscribed to the Google Groups
 Algorithm Geeks group.
 To post to this group, send email to algogeeks@googlegroups.com.
 To unsubscribe from this group, send email to
 algogeeks+unsubscr...@googlegroups.com.
 For more options, visit this group at
 http://groups.google.com/group/algogeeks?hl=en.




-- 
**Regards
SAGAR PAREEK
COMPUTER SCIENCE AND ENGINEERING
NIT ALLAHABAD

-- 
You received this message because you are subscribed to the Google Groups 
Algorithm Geeks group.
To post to this group, send email to algogeeks@googlegroups.com.
To unsubscribe from this group, send email to 
algogeeks+unsubscr...@googlegroups.com.
For more options, visit this group at 
http://groups.google.com/group/algogeeks?hl=en.



[algogeeks] Re: INFINITY

2011-07-19 Thread SAMMM
Yaa guys it hav something to do with UNICODE ... I know that 



On Jul 19, 3:32 pm, sagar pareek sagarpar...@gmail.com wrote:
 yeah in my ubuntu too its not printing  :)





 On Tue, Jul 19, 2011 at 3:20 PM, SAMMM somnath.nit...@gmail.com wrote:
  But if we use gcc or g++ . In that case it doesn't print it .. Wht abt
  tht ...

  On Jul 19, 2:40 pm, Piyush Sinha ecstasy.piy...@gmail.com wrote:
   In Dev C it does

   On Tue, Jul 19, 2011 at 3:08 PM, SAMMM somnath.nit...@gmail.com wrote:
It doesn't display the infinity symbol.

On Jul 19, 2:24 pm, Piyush Sinha ecstasy.piy...@gmail.com wrote:
 *printf(%c\n,236);*

 On Tue, Jul 19, 2011 at 2:45 PM, SAMMM somnath.nit...@gmail.com
  wrote:
  Print the symbol ∞  (INFINITY) through code . Unicode ..

  --
  You received this message because you are subscribed to the Google
Groups
  Algorithm Geeks group.
  To post to this group, send email to algogeeks@googlegroups.com.
  To unsubscribe from this group, send email to
  algogeeks+unsubscr...@googlegroups.com.
  For more options, visit this group at
 http://groups.google.com/group/algogeeks?hl=en.

 --
 *Piyush Sinha*
 *IIIT, Allahabad*
 *+91-7483122727*
 * https://www.facebook.com/profile.php?id=10655377926 NEVER
  SAY
 NEVER
 *

--
You received this message because you are subscribed to the Google
  Groups
Algorithm Geeks group.
To post to this group, send email to algogeeks@googlegroups.com.
To unsubscribe from this group, send email to
algogeeks+unsubscr...@googlegroups.com.
For more options, visit this group at
   http://groups.google.com/group/algogeeks?hl=en.

   --
   *Piyush Sinha*
   *IIIT, Allahabad*
   *+91-7483122727*
   * https://www.facebook.com/profile.php?id=10655377926 NEVER SAY
   NEVER
   *- Hide quoted text -

   - Show quoted text -

  --
  You received this message because you are subscribed to the Google Groups
  Algorithm Geeks group.
  To post to this group, send email to algogeeks@googlegroups.com.
  To unsubscribe from this group, send email to
  algogeeks+unsubscr...@googlegroups.com.
  For more options, visit this group at
 http://groups.google.com/group/algogeeks?hl=en.

 --
 **Regards
 SAGAR PAREEK
 COMPUTER SCIENCE AND ENGINEERING
 NIT ALLAHABAD- Hide quoted text -

 - Show quoted text -

-- 
You received this message because you are subscribed to the Google Groups 
Algorithm Geeks group.
To post to this group, send email to algogeeks@googlegroups.com.
To unsubscribe from this group, send email to 
algogeeks+unsubscr...@googlegroups.com.
For more options, visit this group at 
http://groups.google.com/group/algogeeks?hl=en.



Re: [algogeeks] Solve it

2011-07-19 Thread sagar pareek
would u please code it for me :)

On Tue, Jul 19, 2011 at 3:49 PM, Piyush Sinha ecstasy.piy...@gmail.comwrote:

 I hope it can be solved using DP...check my algo below and give any counter
 case if you get it...

 1. Make an array S equal to the length of the given array where
 S[0] = a[0] and S[1] = max(a[0],a[1])

 2. for i:2 to n-1
  S[i] = max(S[i-2]+a[i], S[i-1])

 3. return S[n-1]

 Hope the above algo works...

 On Tue, Jul 19, 2011 at 2:59 PM, sagar pareek sagarpar...@gmail.comwrote:

 Given an array all of whose elements are positive numbesr, find the
 maximum sum of a subsequence with the constraint that no 2 numbers in the
 sequence should be adjecent in the array.

 eg:-
 3 2 7 10 should return (sum of 3 and 10)
 3 2 5 10 7 should returnn 15 (sum of 3,5,7)

 --
 **Regards
 SAGAR PAREEK
 COMPUTER SCIENCE AND ENGINEERING
 NIT ALLAHABAD

  --
 You received this message because you are subscribed to the Google Groups
 Algorithm Geeks group.
 To post to this group, send email to algogeeks@googlegroups.com.
 To unsubscribe from this group, send email to
 algogeeks+unsubscr...@googlegroups.com.
 For more options, visit this group at
 http://groups.google.com/group/algogeeks?hl=en.




 --
 *Piyush Sinha*
 *IIIT, Allahabad*
 *+91-7483122727*
 * https://www.facebook.com/profile.php?id=10655377926 NEVER SAY
 NEVER
 *

  --
 You received this message because you are subscribed to the Google Groups
 Algorithm Geeks group.
 To post to this group, send email to algogeeks@googlegroups.com.
 To unsubscribe from this group, send email to
 algogeeks+unsubscr...@googlegroups.com.
 For more options, visit this group at
 http://groups.google.com/group/algogeeks?hl=en.




-- 
**Regards
SAGAR PAREEK
COMPUTER SCIENCE AND ENGINEERING
NIT ALLAHABAD

-- 
You received this message because you are subscribed to the Google Groups 
Algorithm Geeks group.
To post to this group, send email to algogeeks@googlegroups.com.
To unsubscribe from this group, send email to 
algogeeks+unsubscr...@googlegroups.com.
For more options, visit this group at 
http://groups.google.com/group/algogeeks?hl=en.



Re: [algogeeks] Re: amazon

2011-07-19 Thread Yogesh Yadav
Part 1:
Take 2 pointers moving at different speed...

1st with head-next
2nd with head-next-next

when 2nd will reach the end 1st will be pointing middle... so one result is
obtained

Part 2:
when 1st result is obtained then start with the node pointed by 1st as it
will be on 1/2th part of the list Start 2 pointers again

3rd with head-next
4th with head-next-next

when 4th will reach end point then 3rd will be pointing the node present at
3/4th of the list







On Tue, Jul 19, 2011 at 3:54 PM, surender sanke surend...@gmail.com wrote:

 take two ptrs ptr1 and ptr2 pointing to head
 move ptr1 until 1/4th of size of list.
 move ptr1 and ptr2 until ptr1=null
 ptr2 is pointing at 3/4th

 surender

 On Tue, Jul 19, 2011 at 3:42 PM, SAMMM somnath.nit...@gmail.com wrote:

 Yaa this will work , you need to handle the case for odd number of
 nodes .
 For even number of node it will serve the purpose .

 Yaa for the second part also you can use the ratio concept

 fast pointer : slow pointer = 4:3

 slow = ptr-next-next-next
 fast = ptr-next-next-next-next


 Here also we need to handle the case if the number of node is not a
 multiple of 4 .

 --
 You received this message because you are subscribed to the Google Groups
 Algorithm Geeks group.
 To post to this group, send email to algogeeks@googlegroups.com.
 To unsubscribe from this group, send email to
 algogeeks+unsubscr...@googlegroups.com.
 For more options, visit this group at
 http://groups.google.com/group/algogeeks?hl=en.


  --
 You received this message because you are subscribed to the Google Groups
 Algorithm Geeks group.
 To post to this group, send email to algogeeks@googlegroups.com.
 To unsubscribe from this group, send email to
 algogeeks+unsubscr...@googlegroups.com.
 For more options, visit this group at
 http://groups.google.com/group/algogeeks?hl=en.


-- 
You received this message because you are subscribed to the Google Groups 
Algorithm Geeks group.
To post to this group, send email to algogeeks@googlegroups.com.
To unsubscribe from this group, send email to 
algogeeks+unsubscr...@googlegroups.com.
For more options, visit this group at 
http://groups.google.com/group/algogeeks?hl=en.



Re: [algogeeks] Re: amazon

2011-07-19 Thread Manikanta Babu
Take two pointers ptr1 and ptr2,

ptr1 should move 4 nodes at a time
ptr2 should move 3 nodes at a time,
thats it when the ptr1 reaches the end the ptr2 will be pointing to 3/4th,
same way for m/n th node. But this works best for even number of nodes in
list.

For odd numbers we need to compromise like finding the middle node :)

Cheers,
Mani

On Tue, Jul 19, 2011 at 4:20 PM, Yogesh Yadav medu...@gmail.com wrote:

 Part 1:
 Take 2 pointers moving at different speed...

 1st with head-next
 2nd with head-next-next

 when 2nd will reach the end 1st will be pointing middle... so one result is
 obtained

 Part 2:
 when 1st result is obtained then start with the node pointed by 1st as it
 will be on 1/2th part of the list Start 2 pointers again

 3rd with head-next
 4th with head-next-next

 when 4th will reach end point then 3rd will be pointing the node present at
 3/4th of the list








 On Tue, Jul 19, 2011 at 3:54 PM, surender sanke surend...@gmail.comwrote:

 take two ptrs ptr1 and ptr2 pointing to head
 move ptr1 until 1/4th of size of list.
 move ptr1 and ptr2 until ptr1=null
 ptr2 is pointing at 3/4th

 surender

 On Tue, Jul 19, 2011 at 3:42 PM, SAMMM somnath.nit...@gmail.com wrote:

 Yaa this will work , you need to handle the case for odd number of
 nodes .
 For even number of node it will serve the purpose .

 Yaa for the second part also you can use the ratio concept

 fast pointer : slow pointer = 4:3

 slow = ptr-next-next-next
 fast = ptr-next-next-next-next


 Here also we need to handle the case if the number of node is not a
 multiple of 4 .

 --
 You received this message because you are subscribed to the Google Groups
 Algorithm Geeks group.
 To post to this group, send email to algogeeks@googlegroups.com.
 To unsubscribe from this group, send email to
 algogeeks+unsubscr...@googlegroups.com.
 For more options, visit this group at
 http://groups.google.com/group/algogeeks?hl=en.


  --
 You received this message because you are subscribed to the Google Groups
 Algorithm Geeks group.
 To post to this group, send email to algogeeks@googlegroups.com.
 To unsubscribe from this group, send email to
 algogeeks+unsubscr...@googlegroups.com.
 For more options, visit this group at
 http://groups.google.com/group/algogeeks?hl=en.


  --
 You received this message because you are subscribed to the Google Groups
 Algorithm Geeks group.
 To post to this group, send email to algogeeks@googlegroups.com.
 To unsubscribe from this group, send email to
 algogeeks+unsubscr...@googlegroups.com.
 For more options, visit this group at
 http://groups.google.com/group/algogeeks?hl=en.




-- 
Thanks  Regards,
Mani
http://www.sanidapa.com - The music Search engine

-- 
You received this message because you are subscribed to the Google Groups 
Algorithm Geeks group.
To post to this group, send email to algogeeks@googlegroups.com.
To unsubscribe from this group, send email to 
algogeeks+unsubscr...@googlegroups.com.
For more options, visit this group at 
http://groups.google.com/group/algogeeks?hl=en.



Re: [algogeeks] Solve it

2011-07-19 Thread sagar pareek
Piyush
sorry dude but this will not work

say original array be
6  8  4  1  2  3
then ur new array be
6  8 10 10 12 13 //but original answer is 12

On Tue, Jul 19, 2011 at 3:49 PM, Piyush Sinha ecstasy.piy...@gmail.comwrote:

 I hope it can be solved using DP...check my algo below and give any counter
 case if you get it...

 1. Make an array S equal to the length of the given array where
 S[0] = a[0] and S[1] = max(a[0],a[1])

 2. for i:2 to n-1
  S[i] = max(S[i-2]+a[i], S[i-1])

 3. return S[n-1]

 Hope the above algo works...

 On Tue, Jul 19, 2011 at 2:59 PM, sagar pareek sagarpar...@gmail.comwrote:

 Given an array all of whose elements are positive numbesr, find the
 maximum sum of a subsequence with the constraint that no 2 numbers in the
 sequence should be adjecent in the array.

 eg:-
 3 2 7 10 should return (sum of 3 and 10)
 3 2 5 10 7 should returnn 15 (sum of 3,5,7)

 --
 **Regards
 SAGAR PAREEK
 COMPUTER SCIENCE AND ENGINEERING
 NIT ALLAHABAD

  --
 You received this message because you are subscribed to the Google Groups
 Algorithm Geeks group.
 To post to this group, send email to algogeeks@googlegroups.com.
 To unsubscribe from this group, send email to
 algogeeks+unsubscr...@googlegroups.com.
 For more options, visit this group at
 http://groups.google.com/group/algogeeks?hl=en.




 --
 *Piyush Sinha*
 *IIIT, Allahabad*
 *+91-7483122727*
 * https://www.facebook.com/profile.php?id=10655377926 NEVER SAY
 NEVER
 *

  --
 You received this message because you are subscribed to the Google Groups
 Algorithm Geeks group.
 To post to this group, send email to algogeeks@googlegroups.com.
 To unsubscribe from this group, send email to
 algogeeks+unsubscr...@googlegroups.com.
 For more options, visit this group at
 http://groups.google.com/group/algogeeks?hl=en.




-- 
**Regards
SAGAR PAREEK
COMPUTER SCIENCE AND ENGINEERING
NIT ALLAHABAD

-- 
You received this message because you are subscribed to the Google Groups 
Algorithm Geeks group.
To post to this group, send email to algogeeks@googlegroups.com.
To unsubscribe from this group, send email to 
algogeeks+unsubscr...@googlegroups.com.
For more options, visit this group at 
http://groups.google.com/group/algogeeks?hl=en.



Re: [algogeeks] Re: amazon

2011-07-19 Thread shilpa gupta
ok guys.. i got it thanks to all.

On Tue, Jul 19, 2011 at 4:23 PM, Manikanta Babu
manikantabab...@gmail.comwrote:

 Take two pointers ptr1 and ptr2,

 ptr1 should move 4 nodes at a time
 ptr2 should move 3 nodes at a time,
 thats it when the ptr1 reaches the end the ptr2 will be pointing to 3/4th,
 same way for m/n th node. But this works best for even number of nodes in
 list.

 For odd numbers we need to compromise like finding the middle node :)

 Cheers,
 Mani

 On Tue, Jul 19, 2011 at 4:20 PM, Yogesh Yadav medu...@gmail.com wrote:

 Part 1:
 Take 2 pointers moving at different speed...

 1st with head-next
 2nd with head-next-next

 when 2nd will reach the end 1st will be pointing middle... so one result
 is obtained

 Part 2:
 when 1st result is obtained then start with the node pointed by 1st as it
 will be on 1/2th part of the list Start 2 pointers again

 3rd with head-next
 4th with head-next-next

 when 4th will reach end point then 3rd will be pointing the node present
 at 3/4th of the list








 On Tue, Jul 19, 2011 at 3:54 PM, surender sanke surend...@gmail.comwrote:

 take two ptrs ptr1 and ptr2 pointing to head
 move ptr1 until 1/4th of size of list.
 move ptr1 and ptr2 until ptr1=null
 ptr2 is pointing at 3/4th

 surender

 On Tue, Jul 19, 2011 at 3:42 PM, SAMMM somnath.nit...@gmail.com wrote:

 Yaa this will work , you need to handle the case for odd number of
 nodes .
 For even number of node it will serve the purpose .

 Yaa for the second part also you can use the ratio concept

 fast pointer : slow pointer = 4:3

 slow = ptr-next-next-next
 fast = ptr-next-next-next-next


 Here also we need to handle the case if the number of node is not a
 multiple of 4 .

 --
 You received this message because you are subscribed to the Google
 Groups Algorithm Geeks group.
 To post to this group, send email to algogeeks@googlegroups.com.
 To unsubscribe from this group, send email to
 algogeeks+unsubscr...@googlegroups.com.
 For more options, visit this group at
 http://groups.google.com/group/algogeeks?hl=en.


  --
 You received this message because you are subscribed to the Google Groups
 Algorithm Geeks group.
 To post to this group, send email to algogeeks@googlegroups.com.
 To unsubscribe from this group, send email to
 algogeeks+unsubscr...@googlegroups.com.
 For more options, visit this group at
 http://groups.google.com/group/algogeeks?hl=en.


  --
 You received this message because you are subscribed to the Google Groups
 Algorithm Geeks group.
 To post to this group, send email to algogeeks@googlegroups.com.
 To unsubscribe from this group, send email to
 algogeeks+unsubscr...@googlegroups.com.
 For more options, visit this group at
 http://groups.google.com/group/algogeeks?hl=en.




 --
 Thanks  Regards,
 Mani
 http://www.sanidapa.com - The music Search engine

 --
 You received this message because you are subscribed to the Google Groups
 Algorithm Geeks group.
 To post to this group, send email to algogeeks@googlegroups.com.
 To unsubscribe from this group, send email to
 algogeeks+unsubscr...@googlegroups.com.
 For more options, visit this group at
 http://groups.google.com/group/algogeeks?hl=en.




-- 
Shilpa Gupta

B.Tech. 3rd year
Computer Science and Engineering
MNNIT Allahabad

-- 
You received this message because you are subscribed to the Google Groups 
Algorithm Geeks group.
To post to this group, send email to algogeeks@googlegroups.com.
To unsubscribe from this group, send email to 
algogeeks+unsubscr...@googlegroups.com.
For more options, visit this group at 
http://groups.google.com/group/algogeeks?hl=en.



Re: [algogeeks] Re: amazon

2011-07-19 Thread Yogesh Yadav
@Manikanta :

 assume we have to find 5/60th part of node then

we will take a pointer moving with 5 nodes at a time and 2nd with 60 nodes
at a time...

for pointer with 60 nodes at a time we have to check(head-next!=null) then
(head-next-next!=null) upto 60 nodes...

Don't you think coding part will become much more complicated...




On Tue, Jul 19, 2011 at 4:23 PM, Manikanta Babu
manikantabab...@gmail.comwrote:

 Take two pointers ptr1 and ptr2,

 ptr1 should move 4 nodes at a time
 ptr2 should move 3 nodes at a time,
 thats it when the ptr1 reaches the end the ptr2 will be pointing to 3/4th,
 same way for m/n th node. But this works best for even number of nodes in
 list.

 For odd numbers we need to compromise like finding the middle node :)

 Cheers,
 Mani

 On Tue, Jul 19, 2011 at 4:20 PM, Yogesh Yadav medu...@gmail.com wrote:

 Part 1:
 Take 2 pointers moving at different speed...

 1st with head-next
 2nd with head-next-next

 when 2nd will reach the end 1st will be pointing middle... so one result
 is obtained

 Part 2:
 when 1st result is obtained then start with the node pointed by 1st as it
 will be on 1/2th part of the list Start 2 pointers again

 3rd with head-next
 4th with head-next-next

 when 4th will reach end point then 3rd will be pointing the node present
 at 3/4th of the list








 On Tue, Jul 19, 2011 at 3:54 PM, surender sanke surend...@gmail.comwrote:

 take two ptrs ptr1 and ptr2 pointing to head
 move ptr1 until 1/4th of size of list.
 move ptr1 and ptr2 until ptr1=null
 ptr2 is pointing at 3/4th

 surender

 On Tue, Jul 19, 2011 at 3:42 PM, SAMMM somnath.nit...@gmail.com wrote:

 Yaa this will work , you need to handle the case for odd number of
 nodes .
 For even number of node it will serve the purpose .

 Yaa for the second part also you can use the ratio concept

 fast pointer : slow pointer = 4:3

 slow = ptr-next-next-next
 fast = ptr-next-next-next-next


 Here also we need to handle the case if the number of node is not a
 multiple of 4 .

 --
 You received this message because you are subscribed to the Google
 Groups Algorithm Geeks group.
 To post to this group, send email to algogeeks@googlegroups.com.
 To unsubscribe from this group, send email to
 algogeeks+unsubscr...@googlegroups.com.
 For more options, visit this group at
 http://groups.google.com/group/algogeeks?hl=en.


  --
 You received this message because you are subscribed to the Google Groups
 Algorithm Geeks group.
 To post to this group, send email to algogeeks@googlegroups.com.
 To unsubscribe from this group, send email to
 algogeeks+unsubscr...@googlegroups.com.
 For more options, visit this group at
 http://groups.google.com/group/algogeeks?hl=en.


  --
 You received this message because you are subscribed to the Google Groups
 Algorithm Geeks group.
 To post to this group, send email to algogeeks@googlegroups.com.
 To unsubscribe from this group, send email to
 algogeeks+unsubscr...@googlegroups.com.
 For more options, visit this group at
 http://groups.google.com/group/algogeeks?hl=en.




 --
 Thanks  Regards,
 Mani
 http://www.sanidapa.com - The music Search engine

 --
 You received this message because you are subscribed to the Google Groups
 Algorithm Geeks group.
 To post to this group, send email to algogeeks@googlegroups.com.
 To unsubscribe from this group, send email to
 algogeeks+unsubscr...@googlegroups.com.
 For more options, visit this group at
 http://groups.google.com/group/algogeeks?hl=en.


-- 
You received this message because you are subscribed to the Google Groups 
Algorithm Geeks group.
To post to this group, send email to algogeeks@googlegroups.com.
To unsubscribe from this group, send email to 
algogeeks+unsubscr...@googlegroups.com.
For more options, visit this group at 
http://groups.google.com/group/algogeeks?hl=en.



Re: [algogeeks] Solve it

2011-07-19 Thread Nitish Garg
The answer to the test case you mentioned is 13 only, 6+4+3 = 13.
Piyush's solution will do it.

On Tue, Jul 19, 2011 at 4:30 PM, sagar pareek sagarpar...@gmail.com wrote:

 Piyush
 sorry dude but this will not work

 say original array be
 6  8  4  1  2  3
 then ur new array be
 6  8 10 10 12 13 //but original answer is 12

 On Tue, Jul 19, 2011 at 3:49 PM, Piyush Sinha ecstasy.piy...@gmail.comwrote:

 I hope it can be solved using DP...check my algo below and give any
 counter case if you get it...

 1. Make an array S equal to the length of the given array where
 S[0] = a[0] and S[1] = max(a[0],a[1])

 2. for i:2 to n-1
  S[i] = max(S[i-2]+a[i], S[i-1])

 3. return S[n-1]

 Hope the above algo works...

 On Tue, Jul 19, 2011 at 2:59 PM, sagar pareek sagarpar...@gmail.comwrote:

 Given an array all of whose elements are positive numbesr, find the
 maximum sum of a subsequence with the constraint that no 2 numbers in the
 sequence should be adjecent in the array.

 eg:-
 3 2 7 10 should return (sum of 3 and 10)
 3 2 5 10 7 should returnn 15 (sum of 3,5,7)

 --
 **Regards
 SAGAR PAREEK
 COMPUTER SCIENCE AND ENGINEERING
 NIT ALLAHABAD

  --
 You received this message because you are subscribed to the Google Groups
 Algorithm Geeks group.
 To post to this group, send email to algogeeks@googlegroups.com.
 To unsubscribe from this group, send email to
 algogeeks+unsubscr...@googlegroups.com.
 For more options, visit this group at
 http://groups.google.com/group/algogeeks?hl=en.




 --
 *Piyush Sinha*
 *IIIT, Allahabad*
 *+91-7483122727*
 * https://www.facebook.com/profile.php?id=10655377926 NEVER SAY
 NEVER
 *

  --
 You received this message because you are subscribed to the Google Groups
 Algorithm Geeks group.
 To post to this group, send email to algogeeks@googlegroups.com.
 To unsubscribe from this group, send email to
 algogeeks+unsubscr...@googlegroups.com.
 For more options, visit this group at
 http://groups.google.com/group/algogeeks?hl=en.




 --
 **Regards
 SAGAR PAREEK
 COMPUTER SCIENCE AND ENGINEERING
 NIT ALLAHABAD

  --
 You received this message because you are subscribed to the Google Groups
 Algorithm Geeks group.
 To post to this group, send email to algogeeks@googlegroups.com.
 To unsubscribe from this group, send email to
 algogeeks+unsubscr...@googlegroups.com.
 For more options, visit this group at
 http://groups.google.com/group/algogeeks?hl=en.


-- 
You received this message because you are subscribed to the Google Groups 
Algorithm Geeks group.
To post to this group, send email to algogeeks@googlegroups.com.
To unsubscribe from this group, send email to 
algogeeks+unsubscr...@googlegroups.com.
For more options, visit this group at 
http://groups.google.com/group/algogeeks?hl=en.



Re: [algogeeks] Re: amazon

2011-07-19 Thread saurabh singh
Well the creator of programming languages were smart enuf to give loops for
that.:)

On Tue, Jul 19, 2011 at 4:31 PM, Yogesh Yadav medu...@gmail.com wrote:

 @Manikanta :

  assume we have to find 5/60th part of node then

 we will take a pointer moving with 5 nodes at a time and 2nd with 60 nodes
 at a time...

 for pointer with 60 nodes at a time we have to check(head-next!=null) then
 (head-next-next!=null) upto 60 nodes...

 Don't you think coding part will become much more complicated...




 On Tue, Jul 19, 2011 at 4:23 PM, Manikanta Babu manikantabab...@gmail.com
  wrote:

 Take two pointers ptr1 and ptr2,

 ptr1 should move 4 nodes at a time
 ptr2 should move 3 nodes at a time,
 thats it when the ptr1 reaches the end the ptr2 will be pointing to 3/4th,
 same way for m/n th node. But this works best for even number of nodes in
 list.

 For odd numbers we need to compromise like finding the middle node :)

 Cheers,
 Mani

 On Tue, Jul 19, 2011 at 4:20 PM, Yogesh Yadav medu...@gmail.com wrote:

 Part 1:
 Take 2 pointers moving at different speed...

 1st with head-next
 2nd with head-next-next

 when 2nd will reach the end 1st will be pointing middle... so one result
 is obtained

 Part 2:
 when 1st result is obtained then start with the node pointed by 1st as it
 will be on 1/2th part of the list Start 2 pointers again

 3rd with head-next
 4th with head-next-next

 when 4th will reach end point then 3rd will be pointing the node present
 at 3/4th of the list








 On Tue, Jul 19, 2011 at 3:54 PM, surender sanke surend...@gmail.comwrote:

 take two ptrs ptr1 and ptr2 pointing to head
 move ptr1 until 1/4th of size of list.
 move ptr1 and ptr2 until ptr1=null
 ptr2 is pointing at 3/4th

 surender

 On Tue, Jul 19, 2011 at 3:42 PM, SAMMM somnath.nit...@gmail.comwrote:

 Yaa this will work , you need to handle the case for odd number of
 nodes .
 For even number of node it will serve the purpose .

 Yaa for the second part also you can use the ratio concept

 fast pointer : slow pointer = 4:3

 slow = ptr-next-next-next
 fast = ptr-next-next-next-next


 Here also we need to handle the case if the number of node is not a
 multiple of 4 .

 --
 You received this message because you are subscribed to the Google
 Groups Algorithm Geeks group.
 To post to this group, send email to algogeeks@googlegroups.com.
 To unsubscribe from this group, send email to
 algogeeks+unsubscr...@googlegroups.com.
 For more options, visit this group at
 http://groups.google.com/group/algogeeks?hl=en.


  --
 You received this message because you are subscribed to the Google
 Groups Algorithm Geeks group.
 To post to this group, send email to algogeeks@googlegroups.com.
 To unsubscribe from this group, send email to
 algogeeks+unsubscr...@googlegroups.com.
 For more options, visit this group at
 http://groups.google.com/group/algogeeks?hl=en.


  --
 You received this message because you are subscribed to the Google Groups
 Algorithm Geeks group.
 To post to this group, send email to algogeeks@googlegroups.com.
 To unsubscribe from this group, send email to
 algogeeks+unsubscr...@googlegroups.com.
 For more options, visit this group at
 http://groups.google.com/group/algogeeks?hl=en.




 --
 Thanks  Regards,
 Mani
 http://www.sanidapa.com - The music Search engine

 --
 You received this message because you are subscribed to the Google Groups
 Algorithm Geeks group.
 To post to this group, send email to algogeeks@googlegroups.com.
 To unsubscribe from this group, send email to
 algogeeks+unsubscr...@googlegroups.com.
 For more options, visit this group at
 http://groups.google.com/group/algogeeks?hl=en.


  --
 You received this message because you are subscribed to the Google Groups
 Algorithm Geeks group.
 To post to this group, send email to algogeeks@googlegroups.com.
 To unsubscribe from this group, send email to
 algogeeks+unsubscr...@googlegroups.com.
 For more options, visit this group at
 http://groups.google.com/group/algogeeks?hl=en.




-- 
Saurabh Singh
B.Tech (Computer Science)
MNNIT ALLAHABAD

-- 
You received this message because you are subscribed to the Google Groups 
Algorithm Geeks group.
To post to this group, send email to algogeeks@googlegroups.com.
To unsubscribe from this group, send email to 
algogeeks+unsubscr...@googlegroups.com.
For more options, visit this group at 
http://groups.google.com/group/algogeeks?hl=en.



Re: [algogeeks] Solve it

2011-07-19 Thread Piyush Sinha
I think  6+4+3  6+4+2

On Tue, Jul 19, 2011 at 4:30 PM, sagar pareek sagarpar...@gmail.com wrote:

 Piyush
 sorry dude but this will not work

 say original array be
 6  8  4  1  2  3
 then ur new array be
 6  8 10 10 12 13 //but original answer is 12

 On Tue, Jul 19, 2011 at 3:49 PM, Piyush Sinha ecstasy.piy...@gmail.comwrote:

 I hope it can be solved using DP...check my algo below and give any
 counter case if you get it...

 1. Make an array S equal to the length of the given array where
 S[0] = a[0] and S[1] = max(a[0],a[1])

 2. for i:2 to n-1
  S[i] = max(S[i-2]+a[i], S[i-1])

 3. return S[n-1]

 Hope the above algo works...

 On Tue, Jul 19, 2011 at 2:59 PM, sagar pareek sagarpar...@gmail.comwrote:

 Given an array all of whose elements are positive numbesr, find the
 maximum sum of a subsequence with the constraint that no 2 numbers in the
 sequence should be adjecent in the array.

 eg:-
 3 2 7 10 should return (sum of 3 and 10)
 3 2 5 10 7 should returnn 15 (sum of 3,5,7)

 --
 **Regards
 SAGAR PAREEK
 COMPUTER SCIENCE AND ENGINEERING
 NIT ALLAHABAD

  --
 You received this message because you are subscribed to the Google Groups
 Algorithm Geeks group.
 To post to this group, send email to algogeeks@googlegroups.com.
 To unsubscribe from this group, send email to
 algogeeks+unsubscr...@googlegroups.com.
 For more options, visit this group at
 http://groups.google.com/group/algogeeks?hl=en.




 --
 *Piyush Sinha*
 *IIIT, Allahabad*
 *+91-7483122727*
 * https://www.facebook.com/profile.php?id=10655377926 NEVER SAY
 NEVER
 *

  --
 You received this message because you are subscribed to the Google Groups
 Algorithm Geeks group.
 To post to this group, send email to algogeeks@googlegroups.com.
 To unsubscribe from this group, send email to
 algogeeks+unsubscr...@googlegroups.com.
 For more options, visit this group at
 http://groups.google.com/group/algogeeks?hl=en.




 --
 **Regards
 SAGAR PAREEK
 COMPUTER SCIENCE AND ENGINEERING
 NIT ALLAHABAD

  --
 You received this message because you are subscribed to the Google Groups
 Algorithm Geeks group.
 To post to this group, send email to algogeeks@googlegroups.com.
 To unsubscribe from this group, send email to
 algogeeks+unsubscr...@googlegroups.com.
 For more options, visit this group at
 http://groups.google.com/group/algogeeks?hl=en.




-- 
*Piyush Sinha*
*IIIT, Allahabad*
*+91-7483122727*
* https://www.facebook.com/profile.php?id=10655377926 NEVER SAY
NEVER
*

-- 
You received this message because you are subscribed to the Google Groups 
Algorithm Geeks group.
To post to this group, send email to algogeeks@googlegroups.com.
To unsubscribe from this group, send email to 
algogeeks+unsubscr...@googlegroups.com.
For more options, visit this group at 
http://groups.google.com/group/algogeeks?hl=en.



Re: [algogeeks] Solve it

2011-07-19 Thread sagar pareek
oh yeah my misunderstanding sorry

On Tue, Jul 19, 2011 at 4:33 PM, Piyush Sinha ecstasy.piy...@gmail.comwrote:

 I think  6+4+3  6+4+2


 On Tue, Jul 19, 2011 at 4:30 PM, sagar pareek sagarpar...@gmail.comwrote:

 Piyush
 sorry dude but this will not work

 say original array be
 6  8  4  1  2  3
 then ur new array be
 6  8 10 10 12 13 //but original answer is 12

 On Tue, Jul 19, 2011 at 3:49 PM, Piyush Sinha 
 ecstasy.piy...@gmail.comwrote:

 I hope it can be solved using DP...check my algo below and give any
 counter case if you get it...

 1. Make an array S equal to the length of the given array where
 S[0] = a[0] and S[1] = max(a[0],a[1])

 2. for i:2 to n-1
  S[i] = max(S[i-2]+a[i], S[i-1])

 3. return S[n-1]

 Hope the above algo works...

 On Tue, Jul 19, 2011 at 2:59 PM, sagar pareek sagarpar...@gmail.comwrote:

 Given an array all of whose elements are positive numbesr, find the
 maximum sum of a subsequence with the constraint that no 2 numbers in the
 sequence should be adjecent in the array.

 eg:-
 3 2 7 10 should return (sum of 3 and 10)
 3 2 5 10 7 should returnn 15 (sum of 3,5,7)

 --
 **Regards
 SAGAR PAREEK
 COMPUTER SCIENCE AND ENGINEERING
 NIT ALLAHABAD

  --
 You received this message because you are subscribed to the Google
 Groups Algorithm Geeks group.
 To post to this group, send email to algogeeks@googlegroups.com.
 To unsubscribe from this group, send email to
 algogeeks+unsubscr...@googlegroups.com.
 For more options, visit this group at
 http://groups.google.com/group/algogeeks?hl=en.




 --
 *Piyush Sinha*
 *IIIT, Allahabad*
 *+91-7483122727*
 * https://www.facebook.com/profile.php?id=10655377926 NEVER SAY
 NEVER
 *

  --
 You received this message because you are subscribed to the Google Groups
 Algorithm Geeks group.
 To post to this group, send email to algogeeks@googlegroups.com.
 To unsubscribe from this group, send email to
 algogeeks+unsubscr...@googlegroups.com.
 For more options, visit this group at
 http://groups.google.com/group/algogeeks?hl=en.




 --
 **Regards
 SAGAR PAREEK
 COMPUTER SCIENCE AND ENGINEERING
 NIT ALLAHABAD

  --
 You received this message because you are subscribed to the Google Groups
 Algorithm Geeks group.
 To post to this group, send email to algogeeks@googlegroups.com.
 To unsubscribe from this group, send email to
 algogeeks+unsubscr...@googlegroups.com.
 For more options, visit this group at
 http://groups.google.com/group/algogeeks?hl=en.




 --
 *Piyush Sinha*
 *IIIT, Allahabad*
 *+91-7483122727*
 * https://www.facebook.com/profile.php?id=10655377926 NEVER SAY
 NEVER
 *

  --
 You received this message because you are subscribed to the Google Groups
 Algorithm Geeks group.
 To post to this group, send email to algogeeks@googlegroups.com.
 To unsubscribe from this group, send email to
 algogeeks+unsubscr...@googlegroups.com.
 For more options, visit this group at
 http://groups.google.com/group/algogeeks?hl=en.




-- 
**Regards
SAGAR PAREEK
COMPUTER SCIENCE AND ENGINEERING
NIT ALLAHABAD

-- 
You received this message because you are subscribed to the Google Groups 
Algorithm Geeks group.
To post to this group, send email to algogeeks@googlegroups.com.
To unsubscribe from this group, send email to 
algogeeks+unsubscr...@googlegroups.com.
For more options, visit this group at 
http://groups.google.com/group/algogeeks?hl=en.



Re: [algogeeks] Solve it

2011-07-19 Thread sagar pareek
well thanks for the solution

On Tue, Jul 19, 2011 at 4:34 PM, sagar pareek sagarpar...@gmail.com wrote:

 oh yeah my misunderstanding sorry


 On Tue, Jul 19, 2011 at 4:33 PM, Piyush Sinha ecstasy.piy...@gmail.comwrote:

 I think  6+4+3  6+4+2


 On Tue, Jul 19, 2011 at 4:30 PM, sagar pareek sagarpar...@gmail.comwrote:

 Piyush
 sorry dude but this will not work

 say original array be
 6  8  4  1  2  3
 then ur new array be
 6  8 10 10 12 13 //but original answer is 12

 On Tue, Jul 19, 2011 at 3:49 PM, Piyush Sinha 
 ecstasy.piy...@gmail.comwrote:

 I hope it can be solved using DP...check my algo below and give any
 counter case if you get it...

 1. Make an array S equal to the length of the given array where
 S[0] = a[0] and S[1] = max(a[0],a[1])

 2. for i:2 to n-1
  S[i] = max(S[i-2]+a[i], S[i-1])

 3. return S[n-1]

 Hope the above algo works...

 On Tue, Jul 19, 2011 at 2:59 PM, sagar pareek sagarpar...@gmail.comwrote:

 Given an array all of whose elements are positive numbesr, find the
 maximum sum of a subsequence with the constraint that no 2 numbers in the
 sequence should be adjecent in the array.

 eg:-
 3 2 7 10 should return (sum of 3 and 10)
 3 2 5 10 7 should returnn 15 (sum of 3,5,7)

 --
 **Regards
 SAGAR PAREEK
 COMPUTER SCIENCE AND ENGINEERING
 NIT ALLAHABAD

  --
 You received this message because you are subscribed to the Google
 Groups Algorithm Geeks group.
 To post to this group, send email to algogeeks@googlegroups.com.
 To unsubscribe from this group, send email to
 algogeeks+unsubscr...@googlegroups.com.
 For more options, visit this group at
 http://groups.google.com/group/algogeeks?hl=en.




 --
 *Piyush Sinha*
 *IIIT, Allahabad*
 *+91-7483122727*
 * https://www.facebook.com/profile.php?id=10655377926 NEVER SAY
 NEVER
 *

  --
 You received this message because you are subscribed to the Google
 Groups Algorithm Geeks group.
 To post to this group, send email to algogeeks@googlegroups.com.
 To unsubscribe from this group, send email to
 algogeeks+unsubscr...@googlegroups.com.
 For more options, visit this group at
 http://groups.google.com/group/algogeeks?hl=en.




 --
 **Regards
 SAGAR PAREEK
 COMPUTER SCIENCE AND ENGINEERING
 NIT ALLAHABAD

  --
 You received this message because you are subscribed to the Google Groups
 Algorithm Geeks group.
 To post to this group, send email to algogeeks@googlegroups.com.
 To unsubscribe from this group, send email to
 algogeeks+unsubscr...@googlegroups.com.
 For more options, visit this group at
 http://groups.google.com/group/algogeeks?hl=en.




 --
 *Piyush Sinha*
 *IIIT, Allahabad*
 *+91-7483122727*
 * https://www.facebook.com/profile.php?id=10655377926 NEVER SAY
 NEVER
 *

  --
 You received this message because you are subscribed to the Google Groups
 Algorithm Geeks group.
 To post to this group, send email to algogeeks@googlegroups.com.
 To unsubscribe from this group, send email to
 algogeeks+unsubscr...@googlegroups.com.
 For more options, visit this group at
 http://groups.google.com/group/algogeeks?hl=en.




 --
 **Regards
 SAGAR PAREEK
 COMPUTER SCIENCE AND ENGINEERING
 NIT ALLAHABAD




-- 
**Regards
SAGAR PAREEK
COMPUTER SCIENCE AND ENGINEERING
NIT ALLAHABAD

-- 
You received this message because you are subscribed to the Google Groups 
Algorithm Geeks group.
To post to this group, send email to algogeeks@googlegroups.com.
To unsubscribe from this group, send email to 
algogeeks+unsubscr...@googlegroups.com.
For more options, visit this group at 
http://groups.google.com/group/algogeeks?hl=en.



Re: [algogeeks] Solve it

2011-07-19 Thread ankit sambyal
@Sagar: 13 is the correct answer. (6+4+3)

-- 
You received this message because you are subscribed to the Google Groups 
Algorithm Geeks group.
To post to this group, send email to algogeeks@googlegroups.com.
To unsubscribe from this group, send email to 
algogeeks+unsubscr...@googlegroups.com.
For more options, visit this group at 
http://groups.google.com/group/algogeeks?hl=en.



Re: [algogeeks] Solve it

2011-07-19 Thread sagar pareek
ok ok ok thank you all

On Tue, Jul 19, 2011 at 4:35 PM, ankit sambyal ankitsamb...@gmail.comwrote:

 @Sagar: 13 is the correct answer. (6+4+3)

 --
 You received this message because you are subscribed to the Google Groups
 Algorithm Geeks group.
 To post to this group, send email to algogeeks@googlegroups.com.
 To unsubscribe from this group, send email to
 algogeeks+unsubscr...@googlegroups.com.
 For more options, visit this group at
 http://groups.google.com/group/algogeeks?hl=en.




-- 
**Regards
SAGAR PAREEK
COMPUTER SCIENCE AND ENGINEERING
NIT ALLAHABAD

-- 
You received this message because you are subscribed to the Google Groups 
Algorithm Geeks group.
To post to this group, send email to algogeeks@googlegroups.com.
To unsubscribe from this group, send email to 
algogeeks+unsubscr...@googlegroups.com.
For more options, visit this group at 
http://groups.google.com/group/algogeeks?hl=en.



Re: [algogeeks] Solve it

2011-07-19 Thread oppilas .
New constraint:-
What if the array also contains positive and negative numbers?

On Tue, Jul 19, 2011 at 4:36 PM, sagar pareek sagarpar...@gmail.com wrote:

 ok ok ok thank you all


 On Tue, Jul 19, 2011 at 4:35 PM, ankit sambyal ankitsamb...@gmail.comwrote:

 @Sagar: 13 is the correct answer. (6+4+3)

 --
 You received this message because you are subscribed to the Google Groups
 Algorithm Geeks group.
 To post to this group, send email to algogeeks@googlegroups.com.
 To unsubscribe from this group, send email to
 algogeeks+unsubscr...@googlegroups.com.
 For more options, visit this group at
 http://groups.google.com/group/algogeeks?hl=en.




 --
 Regards
 SAGAR PAREEK
 COMPUTER SCIENCE AND ENGINEERING
 NIT ALLAHABAD

 --
 You received this message because you are subscribed to the Google Groups
 Algorithm Geeks group.
 To post to this group, send email to algogeeks@googlegroups.com.
 To unsubscribe from this group, send email to
 algogeeks+unsubscr...@googlegroups.com.
 For more options, visit this group at
 http://groups.google.com/group/algogeeks?hl=en.


-- 
You received this message because you are subscribed to the Google Groups 
Algorithm Geeks group.
To post to this group, send email to algogeeks@googlegroups.com.
To unsubscribe from this group, send email to 
algogeeks+unsubscr...@googlegroups.com.
For more options, visit this group at 
http://groups.google.com/group/algogeeks?hl=en.



Re: [algogeeks] Solve it

2011-07-19 Thread Nitish Garg
Won't this recurrence work:
s[i] = max(s[i-2], s[i-2]+a[i], a[i-1]) work?
I think it works.

On Tue, Jul 19, 2011 at 4:54 PM, oppilas . jatka.oppimi...@gmail.comwrote:

 New constraint:-
 What if the array also contains positive and negative numbers?

 On Tue, Jul 19, 2011 at 4:36 PM, sagar pareek sagarpar...@gmail.comwrote:

 ok ok ok thank you all


 On Tue, Jul 19, 2011 at 4:35 PM, ankit sambyal ankitsamb...@gmail.comwrote:

 @Sagar: 13 is the correct answer. (6+4+3)

 --
 You received this message because you are subscribed to the Google Groups
 Algorithm Geeks group.
 To post to this group, send email to algogeeks@googlegroups.com.
 To unsubscribe from this group, send email to
 algogeeks+unsubscr...@googlegroups.com.
 For more options, visit this group at
 http://groups.google.com/group/algogeeks?hl=en.




 --
 Regards
 SAGAR PAREEK
 COMPUTER SCIENCE AND ENGINEERING
 NIT ALLAHABAD

 --
 You received this message because you are subscribed to the Google Groups
 Algorithm Geeks group.
 To post to this group, send email to algogeeks@googlegroups.com.
 To unsubscribe from this group, send email to
 algogeeks+unsubscr...@googlegroups.com.
 For more options, visit this group at
 http://groups.google.com/group/algogeeks?hl=en.


  --
 You received this message because you are subscribed to the Google Groups
 Algorithm Geeks group.
 To post to this group, send email to algogeeks@googlegroups.com.
 To unsubscribe from this group, send email to
 algogeeks+unsubscr...@googlegroups.com.
 For more options, visit this group at
 http://groups.google.com/group/algogeeks?hl=en.


-- 
You received this message because you are subscribed to the Google Groups 
Algorithm Geeks group.
To post to this group, send email to algogeeks@googlegroups.com.
To unsubscribe from this group, send email to 
algogeeks+unsubscr...@googlegroups.com.
For more options, visit this group at 
http://groups.google.com/group/algogeeks?hl=en.



Re: [algogeeks] Memory Allocation

2011-07-19 Thread Abhi
Yes, to a great extent. Thanks :)
Obviously we can pass arrays across functions by declaring them static. Just 
saying.

-- 
You received this message because you are subscribed to the Google Groups 
Algorithm Geeks group.
To view this discussion on the web visit 
https://groups.google.com/d/msg/algogeeks/-/dpYbwj0LyhcJ.
To post to this group, send email to algogeeks@googlegroups.com.
To unsubscribe from this group, send email to 
algogeeks+unsubscr...@googlegroups.com.
For more options, visit this group at 
http://groups.google.com/group/algogeeks?hl=en.



Re: [algogeeks] Solve it

2011-07-19 Thread Nitish Garg
Typo in my above post.
s[i] = max(s[i-2], s[i-2]+a[i], s[i-1]) work?

On Tue, Jul 19, 2011 at 4:59 PM, Nitish Garg nitishgarg1...@gmail.comwrote:

 Won't this recurrence work:
 s[i] = max(s[i-2], s[i-2]+a[i], a[i-1]) work?
 I think it works.


 On Tue, Jul 19, 2011 at 4:54 PM, oppilas . jatka.oppimi...@gmail.comwrote:

 New constraint:-
 What if the array also contains positive and negative numbers?

 On Tue, Jul 19, 2011 at 4:36 PM, sagar pareek sagarpar...@gmail.comwrote:

 ok ok ok thank you all


 On Tue, Jul 19, 2011 at 4:35 PM, ankit sambyal 
 ankitsamb...@gmail.comwrote:

 @Sagar: 13 is the correct answer. (6+4+3)

 --
 You received this message because you are subscribed to the Google
 Groups Algorithm Geeks group.
 To post to this group, send email to algogeeks@googlegroups.com.
 To unsubscribe from this group, send email to
 algogeeks+unsubscr...@googlegroups.com.
 For more options, visit this group at
 http://groups.google.com/group/algogeeks?hl=en.




 --
 Regards
 SAGAR PAREEK
 COMPUTER SCIENCE AND ENGINEERING
 NIT ALLAHABAD

 --
 You received this message because you are subscribed to the Google Groups
 Algorithm Geeks group.
 To post to this group, send email to algogeeks@googlegroups.com.
 To unsubscribe from this group, send email to
 algogeeks+unsubscr...@googlegroups.com.
 For more options, visit this group at
 http://groups.google.com/group/algogeeks?hl=en.


  --
 You received this message because you are subscribed to the Google Groups
 Algorithm Geeks group.
 To post to this group, send email to algogeeks@googlegroups.com.
 To unsubscribe from this group, send email to
 algogeeks+unsubscr...@googlegroups.com.
 For more options, visit this group at
 http://groups.google.com/group/algogeeks?hl=en.




-- 
You received this message because you are subscribed to the Google Groups 
Algorithm Geeks group.
To post to this group, send email to algogeeks@googlegroups.com.
To unsubscribe from this group, send email to 
algogeeks+unsubscr...@googlegroups.com.
For more options, visit this group at 
http://groups.google.com/group/algogeeks?hl=en.



[algogeeks] GETS

2011-07-19 Thread Ankit Sablok
the gets function gives me error when i execute the following code
plzz provide suggestions and answers

char str[100];

while(n--)
{
  gets(str);
  puts(str);
}

suggest alternative methods to solve the anomaly

-- 
You received this message because you are subscribed to the Google Groups 
Algorithm Geeks group.
To post to this group, send email to algogeeks@googlegroups.com.
To unsubscribe from this group, send email to 
algogeeks+unsubscr...@googlegroups.com.
For more options, visit this group at 
http://groups.google.com/group/algogeeks?hl=en.



[algogeeks] Re: GETS

2011-07-19 Thread SAMMM
You can try this :-

fgets(buff,sizeof(buff),stdin) ..
It is recomended not to use gets in g++ or gcc ..




On Jul 19, 4:40 pm, Ankit Sablok ankitsablok19091...@gmail.com
wrote:
 the gets function gives me error when i execute the following code
 plzz provide suggestions and answers

 char str[100];

 while(n--)
 {
   gets(str);
   puts(str);

 }

 suggest alternative methods to solve the anomaly

-- 
You received this message because you are subscribed to the Google Groups 
Algorithm Geeks group.
To post to this group, send email to algogeeks@googlegroups.com.
To unsubscribe from this group, send email to 
algogeeks+unsubscr...@googlegroups.com.
For more options, visit this group at 
http://groups.google.com/group/algogeeks?hl=en.



Re: [algogeeks] Puzzle[Google] Can be Solved programatically as well

2011-07-19 Thread abhishek iyer
Can you please the explain the approach.. Would be very helpful... I dint
get it ...

On Tue, Jul 19, 2011 at 1:37 PM, archita monga kool.arc...@gmail.comwrote:

 yaa..


 On Tue, Jul 19, 2011 at 1:35 PM, alagammai narayanan 
 alagamma...@gmail.com wrote:

 Yep.. Its German.. Were you guys able to come up with the 5 * 5 matrix...
 As in who lives in which house?What does he drink,smoke etc..


 On Tue, Jul 19, 2011 at 1:31 PM, archita monga kool.arc...@gmail.comwrote:

 German!


 On Tue, Jul 19, 2011 at 1:22 PM, D!leep Gupta 
 dileep.smil...@gmail.comwrote:

 Ans. *German*

 --
 You received this message because you are subscribed to the Google
 Groups Algorithm Geeks group.
 To post to this group, send email to algogeeks@googlegroups.com.
 To unsubscribe from this group, send email to
 algogeeks+unsubscr...@googlegroups.com.
 For more options, visit this group at
 http://groups.google.com/group/algogeeks?hl=en.




 --
 Archita Monga

  --
 You received this message because you are subscribed to the Google Groups
 Algorithm Geeks group.
 To post to this group, send email to algogeeks@googlegroups.com.
 To unsubscribe from this group, send email to
 algogeeks+unsubscr...@googlegroups.com.
 For more options, visit this group at
 http://groups.google.com/group/algogeeks?hl=en.


  --
 You received this message because you are subscribed to the Google Groups
 Algorithm Geeks group.
 To post to this group, send email to algogeeks@googlegroups.com.
 To unsubscribe from this group, send email to
 algogeeks+unsubscr...@googlegroups.com.
 For more options, visit this group at
 http://groups.google.com/group/algogeeks?hl=en.




 --
 Archita Monga

 --
 You received this message because you are subscribed to the Google Groups
 Algorithm Geeks group.
 To post to this group, send email to algogeeks@googlegroups.com.
 To unsubscribe from this group, send email to
 algogeeks+unsubscr...@googlegroups.com.
 For more options, visit this group at
 http://groups.google.com/group/algogeeks?hl=en.




-- 
Thanks  Regards
Abhishek Iyer

If You Obey All the Rules, You Will Miss All the Fun. 

-- 
You received this message because you are subscribed to the Google Groups 
Algorithm Geeks group.
To post to this group, send email to algogeeks@googlegroups.com.
To unsubscribe from this group, send email to 
algogeeks+unsubscr...@googlegroups.com.
For more options, visit this group at 
http://groups.google.com/group/algogeeks?hl=en.



[algogeeks] Re: GETS

2011-07-19 Thread Ankit Sablok
Y is it not recommended to use gets in g++ what is the exact reason
can u tell

On Jul 19, 4:47 pm, SAMMM somnath.nit...@gmail.com wrote:
 You can try this :-

 fgets(buff,sizeof(buff),stdin) ..
 It is recomended not to use gets in g++ or gcc ..

 On Jul 19, 4:40 pm, Ankit Sablok ankitsablok19091...@gmail.com
 wrote:







  the gets function gives me error when i execute the following code
  plzz provide suggestions and answers

  char str[100];

  while(n--)
  {
    gets(str);
    puts(str);

  }

  suggest alternative methods to solve the anomaly

-- 
You received this message because you are subscribed to the Google Groups 
Algorithm Geeks group.
To post to this group, send email to algogeeks@googlegroups.com.
To unsubscribe from this group, send email to 
algogeeks+unsubscr...@googlegroups.com.
For more options, visit this group at 
http://groups.google.com/group/algogeeks?hl=en.



[algogeeks] Re: GETS

2011-07-19 Thread SAMMM
It is because it doesn't check any bound checking . So overflow can
easily occur without user's concent .

On Jul 19, 4:48 pm, Ankit Sablok ankitsablok19091...@gmail.com
wrote:
 Y is it not recommended to use gets in g++ what is the exact reason
 can u tell

 On Jul 19, 4:47 pm, SAMMM somnath.nit...@gmail.com wrote:



  You can try this :-

  fgets(buff,sizeof(buff),stdin) ..
  It is recomended not to use gets in g++ or gcc ..

  On Jul 19, 4:40 pm, Ankit Sablok ankitsablok19091...@gmail.com
  wrote:

   the gets function gives me error when i execute the following code
   plzz provide suggestions and answers

   char str[100];

   while(n--)
   {
     gets(str);
     puts(str);

   }

   suggest alternative methods to solve the anomaly- Hide quoted text -

 - Show quoted text -

-- 
You received this message because you are subscribed to the Google Groups 
Algorithm Geeks group.
To post to this group, send email to algogeeks@googlegroups.com.
To unsubscribe from this group, send email to 
algogeeks+unsubscr...@googlegroups.com.
For more options, visit this group at 
http://groups.google.com/group/algogeeks?hl=en.



Re: [algogeeks] Re: INFINITY

2011-07-19 Thread varun pahwa
printf(\u221E\n);



On Tue, Jul 19, 2011 at 4:10 PM, SAMMM somnath.nit...@gmail.com wrote:

 Yaa guys it hav something to do with UNICODE ... I know that 



 On Jul 19, 3:32 pm, sagar pareek sagarpar...@gmail.com wrote:
  yeah in my ubuntu too its not printing  :)
 
 
 
 
 
  On Tue, Jul 19, 2011 at 3:20 PM, SAMMM somnath.nit...@gmail.com wrote:
   But if we use gcc or g++ . In that case it doesn't print it .. Wht abt
   tht ...
 
   On Jul 19, 2:40 pm, Piyush Sinha ecstasy.piy...@gmail.com wrote:
In Dev C it does
 
On Tue, Jul 19, 2011 at 3:08 PM, SAMMM somnath.nit...@gmail.com
 wrote:
 It doesn't display the infinity symbol.
 
 On Jul 19, 2:24 pm, Piyush Sinha ecstasy.piy...@gmail.com wrote:
  *printf(%c\n,236);*
 
  On Tue, Jul 19, 2011 at 2:45 PM, SAMMM somnath.nit...@gmail.com
 
   wrote:
   Print the symbol ∞  (INFINITY) through code . Unicode ..
 
   --
   You received this message because you are subscribed to the
 Google
 Groups
   Algorithm Geeks group.
   To post to this group, send email to
 algogeeks@googlegroups.com.
   To unsubscribe from this group, send email to
   algogeeks+unsubscr...@googlegroups.com.
   For more options, visit this group at
  http://groups.google.com/group/algogeeks?hl=en.
 
  --
  *Piyush Sinha*
  *IIIT, Allahabad*
  *+91-7483122727*
  * https://www.facebook.com/profile.php?id=10655377926
 NEVER
   SAY
  NEVER
  *
 
 --
 You received this message because you are subscribed to the Google
   Groups
 Algorithm Geeks group.
 To post to this group, send email to algogeeks@googlegroups.com.
 To unsubscribe from this group, send email to
 algogeeks+unsubscr...@googlegroups.com.
 For more options, visit this group at
http://groups.google.com/group/algogeeks?hl=en.
 
--
*Piyush Sinha*
*IIIT, Allahabad*
*+91-7483122727*
* https://www.facebook.com/profile.php?id=10655377926 NEVER
 SAY
NEVER
*- Hide quoted text -
 
- Show quoted text -
 
   --
   You received this message because you are subscribed to the Google
 Groups
   Algorithm Geeks group.
   To post to this group, send email to algogeeks@googlegroups.com.
   To unsubscribe from this group, send email to
   algogeeks+unsubscr...@googlegroups.com.
   For more options, visit this group at
  http://groups.google.com/group/algogeeks?hl=en.
 
  --
  **Regards
  SAGAR PAREEK
  COMPUTER SCIENCE AND ENGINEERING
  NIT ALLAHABAD- Hide quoted text -
 
  - Show quoted text -

 --
 You received this message because you are subscribed to the Google Groups
 Algorithm Geeks group.
 To post to this group, send email to algogeeks@googlegroups.com.
 To unsubscribe from this group, send email to
 algogeeks+unsubscr...@googlegroups.com.
 For more options, visit this group at
 http://groups.google.com/group/algogeeks?hl=en.




-- 
Varun Pahwa
B.Tech (IT)
7th Sem.
Indian Institute of Information Technology Allahabad.
Ph : 09793899112
Official Email :: rit2008...@iiita.ac.in
Another Email :: varunpahwa.ii...@gmail.com

People who fail to plan are those who plan to fail.

-- 
You received this message because you are subscribed to the Google Groups 
Algorithm Geeks group.
To post to this group, send email to algogeeks@googlegroups.com.
To unsubscribe from this group, send email to 
algogeeks+unsubscr...@googlegroups.com.
For more options, visit this group at 
http://groups.google.com/group/algogeeks?hl=en.



Re: [algogeeks] Re: Amazon

2011-07-19 Thread varun pahwa
ankit solution is correct all five bases will be 1/5 and altitude of each
triangle will be h same. so all have equal area. 1/2*(b/5) *h.

On Tue, Jul 19, 2011 at 2:07 PM, sagar pareek sagarpar...@gmail.com wrote:

 Ok then
 What about  ques 2 ?


 On Tue, Jul 19, 2011 at 1:40 PM, Nitish Garg nitishgarg1...@gmail.comwrote:

 The algo gives the number of set bits in the number as pointed out by
 SAMMM above.

 --
 You received this message because you are subscribed to the Google Groups
 Algorithm Geeks group.
 To view this discussion on the web visit
 https://groups.google.com/d/msg/algogeeks/-/lyzigAph1iYJ.

 To post to this group, send email to algogeeks@googlegroups.com.
 To unsubscribe from this group, send email to
 algogeeks+unsubscr...@googlegroups.com.
 For more options, visit this group at
 http://groups.google.com/group/algogeeks?hl=en.




 --
 **Regards
 SAGAR PAREEK
 COMPUTER SCIENCE AND ENGINEERING
 NIT ALLAHABAD

  --
 You received this message because you are subscribed to the Google Groups
 Algorithm Geeks group.
 To post to this group, send email to algogeeks@googlegroups.com.
 To unsubscribe from this group, send email to
 algogeeks+unsubscr...@googlegroups.com.
 For more options, visit this group at
 http://groups.google.com/group/algogeeks?hl=en.




-- 
Varun Pahwa
B.Tech (IT)
7th Sem.
Indian Institute of Information Technology Allahabad.
Ph : 09793899112
Official Email :: rit2008...@iiita.ac.in
Another Email :: varunpahwa.ii...@gmail.com

People who fail to plan are those who plan to fail.

-- 
You received this message because you are subscribed to the Google Groups 
Algorithm Geeks group.
To post to this group, send email to algogeeks@googlegroups.com.
To unsubscribe from this group, send email to 
algogeeks+unsubscr...@googlegroups.com.
For more options, visit this group at 
http://groups.google.com/group/algogeeks?hl=en.



[algogeeks] Re: INFINITY

2011-07-19 Thread SAMMM
It's strange I tried it in GCC version 2.95.3 there is gives error ,
but in upper version it gives the correct answer .
I will thinking it will be same for different version .
I also gone for that , but i was having the intution that it doesn't
give the right answer .

Version is a serious mess.

Moral of the story :--  Keep ur system upgraded .



On Jul 19, 5:24 pm, varun pahwa varunpahwa2...@gmail.com wrote:
 printf(\u221E\n);





 On Tue, Jul 19, 2011 at 4:10 PM, SAMMM somnath.nit...@gmail.com wrote:
  Yaa guys it hav something to do with UNICODE ... I know that 

  On Jul 19, 3:32 pm, sagar pareek sagarpar...@gmail.com wrote:
   yeah in my ubuntu too its not printing  :)

   On Tue, Jul 19, 2011 at 3:20 PM, SAMMM somnath.nit...@gmail.com wrote:
But if we use gcc or g++ . In that case it doesn't print it .. Wht abt
tht ...

On Jul 19, 2:40 pm, Piyush Sinha ecstasy.piy...@gmail.com wrote:
 In Dev C it does

 On Tue, Jul 19, 2011 at 3:08 PM, SAMMM somnath.nit...@gmail.com
  wrote:
  It doesn't display the infinity symbol.

  On Jul 19, 2:24 pm, Piyush Sinha ecstasy.piy...@gmail.com wrote:
   *printf(%c\n,236);*

   On Tue, Jul 19, 2011 at 2:45 PM, SAMMM somnath.nit...@gmail.com

wrote:
Print the symbol ∞  (INFINITY) through code . Unicode ..

--
You received this message because you are subscribed to the
  Google
  Groups
Algorithm Geeks group.
To post to this group, send email to
  algogeeks@googlegroups.com.
To unsubscribe from this group, send email to
algogeeks+unsubscr...@googlegroups.com.
For more options, visit this group at
   http://groups.google.com/group/algogeeks?hl=en.

   --
   *Piyush Sinha*
   *IIIT, Allahabad*
   *+91-7483122727*
   * https://www.facebook.com/profile.php?id=10655377926
  NEVER
SAY
   NEVER
   *

  --
  You received this message because you are subscribed to the Google
Groups
  Algorithm Geeks group.
  To post to this group, send email to algogeeks@googlegroups.com.
  To unsubscribe from this group, send email to
  algogeeks+unsubscr...@googlegroups.com.
  For more options, visit this group at
 http://groups.google.com/group/algogeeks?hl=en.

 --
 *Piyush Sinha*
 *IIIT, Allahabad*
 *+91-7483122727*
 * https://www.facebook.com/profile.php?id=10655377926 NEVER
  SAY
 NEVER
 *- Hide quoted text -

 - Show quoted text -

--
You received this message because you are subscribed to the Google
  Groups
Algorithm Geeks group.
To post to this group, send email to algogeeks@googlegroups.com.
To unsubscribe from this group, send email to
algogeeks+unsubscr...@googlegroups.com.
For more options, visit this group at
   http://groups.google.com/group/algogeeks?hl=en.

   --
   **Regards
   SAGAR PAREEK
   COMPUTER SCIENCE AND ENGINEERING
   NIT ALLAHABAD- Hide quoted text -

   - Show quoted text -

  --
  You received this message because you are subscribed to the Google Groups
  Algorithm Geeks group.
  To post to this group, send email to algogeeks@googlegroups.com.
  To unsubscribe from this group, send email to
  algogeeks+unsubscr...@googlegroups.com.
  For more options, visit this group at
 http://groups.google.com/group/algogeeks?hl=en.

 --
 Varun Pahwa
 B.Tech (IT)
 7th Sem.
 Indian Institute of Information Technology Allahabad.
 Ph : 09793899112
 Official Email :: rit2008...@iiita.ac.in
 Another Email :: varunpahwa.ii...@gmail.com

 People who fail to plan are those who plan to fail.- Hide quoted text -

 - Show quoted text -

-- 
You received this message because you are subscribed to the Google Groups 
Algorithm Geeks group.
To post to this group, send email to algogeeks@googlegroups.com.
To unsubscribe from this group, send email to 
algogeeks+unsubscr...@googlegroups.com.
For more options, visit this group at 
http://groups.google.com/group/algogeeks?hl=en.



Re: [algogeeks] Re: GETS

2011-07-19 Thread sagar pareek
+1 to SAMM

On Tue, Jul 19, 2011 at 5:32 PM, SAMMM somnath.nit...@gmail.com wrote:

 It is because it doesn't check any bound checking . So overflow can
 easily occur without user's concent .

 On Jul 19, 4:48 pm, Ankit Sablok ankitsablok19091...@gmail.com
 wrote:
  Y is it not recommended to use gets in g++ what is the exact reason
  can u tell
 
  On Jul 19, 4:47 pm, SAMMM somnath.nit...@gmail.com wrote:
 
 
 
   You can try this :-
 
   fgets(buff,sizeof(buff),stdin) ..
   It is recomended not to use gets in g++ or gcc ..
 
   On Jul 19, 4:40 pm, Ankit Sablok ankitsablok19091...@gmail.com
   wrote:
 
the gets function gives me error when i execute the following code
plzz provide suggestions and answers
 
char str[100];
 
while(n--)
{
  gets(str);
  puts(str);
 
}
 
suggest alternative methods to solve the anomaly- Hide quoted text -
 
  - Show quoted text -

 --
 You received this message because you are subscribed to the Google Groups
 Algorithm Geeks group.
 To post to this group, send email to algogeeks@googlegroups.com.
 To unsubscribe from this group, send email to
 algogeeks+unsubscr...@googlegroups.com.
 For more options, visit this group at
 http://groups.google.com/group/algogeeks?hl=en.




-- 
**Regards
SAGAR PAREEK
COMPUTER SCIENCE AND ENGINEERING
NIT ALLAHABAD

-- 
You received this message because you are subscribed to the Google Groups 
Algorithm Geeks group.
To post to this group, send email to algogeeks@googlegroups.com.
To unsubscribe from this group, send email to 
algogeeks+unsubscr...@googlegroups.com.
For more options, visit this group at 
http://groups.google.com/group/algogeeks?hl=en.



Re: [algogeeks] amazon

2011-07-19 Thread saurabh singh
Calculate sum'
create heap
extract first k elements.

On Tue, Jul 19, 2011 at 6:31 PM, Rishabh Maurya poofiefoo...@gmail.comwrote:



 find distances of given point P(x,y) with all other points in O(n) and
 build initial Max-Heap of K elements with key as distance calculated.

 Now compare all remaining  points with the root of the heap . If its
 distance is less then key of root then just change the root node and its key
 and perform max-heapify . If not so leave the heap as it is and process next
 remaining point.

 Finally Heap will contain K elements which are closest to point P.

 So in short complexity is  O(NLog(K))

  --
 You received this message because you are subscribed to the Google Groups
 Algorithm Geeks group.
 To post to this group, send email to algogeeks@googlegroups.com.
 To unsubscribe from this group, send email to
 algogeeks+unsubscr...@googlegroups.com.
 For more options, visit this group at
 http://groups.google.com/group/algogeeks?hl=en.




-- 
Saurabh Singh
B.Tech (Computer Science)
MNNIT ALLAHABAD

-- 
You received this message because you are subscribed to the Google Groups 
Algorithm Geeks group.
To post to this group, send email to algogeeks@googlegroups.com.
To unsubscribe from this group, send email to 
algogeeks+unsubscr...@googlegroups.com.
For more options, visit this group at 
http://groups.google.com/group/algogeeks?hl=en.



Re: [algogeeks] Re: INFINITY

2011-07-19 Thread sagar pareek
its give me warning
 warning: universal character names are only valid in C++ and C99
but running...   :)

On Tue, Jul 19, 2011 at 6:15 PM, SAMMM somnath.nit...@gmail.com wrote:

 It's strange I tried it in GCC version 2.95.3 there is gives error ,
 but in upper version it gives the correct answer .
 I will thinking it will be same for different version .
 I also gone for that , but i was having the intution that it doesn't
 give the right answer .

 Version is a serious mess.

 Moral of the story :--  Keep ur system upgraded .



 On Jul 19, 5:24 pm, varun pahwa varunpahwa2...@gmail.com wrote:
  printf(\u221E\n);
 
 
 
 
 
  On Tue, Jul 19, 2011 at 4:10 PM, SAMMM somnath.nit...@gmail.com wrote:
   Yaa guys it hav something to do with UNICODE ... I know that 
 
   On Jul 19, 3:32 pm, sagar pareek sagarpar...@gmail.com wrote:
yeah in my ubuntu too its not printing  :)
 
On Tue, Jul 19, 2011 at 3:20 PM, SAMMM somnath.nit...@gmail.com
 wrote:
 But if we use gcc or g++ . In that case it doesn't print it .. Wht
 abt
 tht ...
 
 On Jul 19, 2:40 pm, Piyush Sinha ecstasy.piy...@gmail.com wrote:
  In Dev C it does
 
  On Tue, Jul 19, 2011 at 3:08 PM, SAMMM somnath.nit...@gmail.com
 
   wrote:
   It doesn't display the infinity symbol.
 
   On Jul 19, 2:24 pm, Piyush Sinha ecstasy.piy...@gmail.com
 wrote:
*printf(%c\n,236);*
 
On Tue, Jul 19, 2011 at 2:45 PM, SAMMM 
 somnath.nit...@gmail.com
 
 wrote:
 Print the symbol ∞  (INFINITY) through code . Unicode ..
 
 --
 You received this message because you are subscribed to the
   Google
   Groups
 Algorithm Geeks group.
 To post to this group, send email to
   algogeeks@googlegroups.com.
 To unsubscribe from this group, send email to
 algogeeks+unsubscr...@googlegroups.com.
 For more options, visit this group at
http://groups.google.com/group/algogeeks?hl=en.
 
--
*Piyush Sinha*
*IIIT, Allahabad*
*+91-7483122727*
* https://www.facebook.com/profile.php?id=10655377926
   NEVER
 SAY
NEVER
*
 
   --
   You received this message because you are subscribed to the
 Google
 Groups
   Algorithm Geeks group.
   To post to this group, send email to
 algogeeks@googlegroups.com.
   To unsubscribe from this group, send email to
   algogeeks+unsubscr...@googlegroups.com.
   For more options, visit this group at
  http://groups.google.com/group/algogeeks?hl=en.
 
  --
  *Piyush Sinha*
  *IIIT, Allahabad*
  *+91-7483122727*
  * https://www.facebook.com/profile.php?id=10655377926
 NEVER
   SAY
  NEVER
  *- Hide quoted text -
 
  - Show quoted text -
 
 --
 You received this message because you are subscribed to the Google
   Groups
 Algorithm Geeks group.
 To post to this group, send email to algogeeks@googlegroups.com.
 To unsubscribe from this group, send email to
 algogeeks+unsubscr...@googlegroups.com.
 For more options, visit this group at
http://groups.google.com/group/algogeeks?hl=en.
 
--
**Regards
SAGAR PAREEK
COMPUTER SCIENCE AND ENGINEERING
NIT ALLAHABAD- Hide quoted text -
 
- Show quoted text -
 
   --
   You received this message because you are subscribed to the Google
 Groups
   Algorithm Geeks group.
   To post to this group, send email to algogeeks@googlegroups.com.
   To unsubscribe from this group, send email to
   algogeeks+unsubscr...@googlegroups.com.
   For more options, visit this group at
  http://groups.google.com/group/algogeeks?hl=en.
 
  --
  Varun Pahwa
  B.Tech (IT)
  7th Sem.
  Indian Institute of Information Technology Allahabad.
  Ph : 09793899112
  Official Email :: rit2008...@iiita.ac.in
  Another Email :: varunpahwa.ii...@gmail.com
 
  People who fail to plan are those who plan to fail.- Hide quoted text -
 
  - Show quoted text -

 --
 You received this message because you are subscribed to the Google Groups
 Algorithm Geeks group.
 To post to this group, send email to algogeeks@googlegroups.com.
 To unsubscribe from this group, send email to
 algogeeks+unsubscr...@googlegroups.com.
 For more options, visit this group at
 http://groups.google.com/group/algogeeks?hl=en.




-- 
**Regards
SAGAR PAREEK
COMPUTER SCIENCE AND ENGINEERING
NIT ALLAHABAD

-- 
You received this message because you are subscribed to the Google Groups 
Algorithm Geeks group.
To post to this group, send email to algogeeks@googlegroups.com.
To unsubscribe from this group, send email to 
algogeeks+unsubscr...@googlegroups.com.
For more options, visit this group at 
http://groups.google.com/group/algogeeks?hl=en.



Re: [algogeeks] Re: Amazon

2011-07-19 Thread sagar pareek
Sorry for my previous post
i got the solution now :)

On Tue, Jul 19, 2011 at 6:29 PM, sagar pareek sagarpar...@gmail.com wrote:

 Yeah i saw that solution but question is to divide the traingle in 5 equals
 *TRIANGLES  *not 5 equal parts


 On Tue, Jul 19, 2011 at 5:57 PM, varun pahwa varunpahwa2...@gmail.comwrote:

 ankit solution is correct all five bases will be 1/5 and altitude of each
 triangle will be h same. so all have equal area. 1/2*(b/5) *h.


 On Tue, Jul 19, 2011 at 2:07 PM, sagar pareek sagarpar...@gmail.comwrote:

 Ok then
 What about  ques 2 ?


 On Tue, Jul 19, 2011 at 1:40 PM, Nitish Garg 
 nitishgarg1...@gmail.comwrote:

 The algo gives the number of set bits in the number as pointed out by
 SAMMM above.

 --
 You received this message because you are subscribed to the Google
 Groups Algorithm Geeks group.
 To view this discussion on the web visit
 https://groups.google.com/d/msg/algogeeks/-/lyzigAph1iYJ.

 To post to this group, send email to algogeeks@googlegroups.com.
 To unsubscribe from this group, send email to
 algogeeks+unsubscr...@googlegroups.com.
 For more options, visit this group at
 http://groups.google.com/group/algogeeks?hl=en.




 --
 **Regards
 SAGAR PAREEK
 COMPUTER SCIENCE AND ENGINEERING
 NIT ALLAHABAD

  --
 You received this message because you are subscribed to the Google Groups
 Algorithm Geeks group.
 To post to this group, send email to algogeeks@googlegroups.com.
 To unsubscribe from this group, send email to
 algogeeks+unsubscr...@googlegroups.com.
 For more options, visit this group at
 http://groups.google.com/group/algogeeks?hl=en.




 --
 Varun Pahwa
 B.Tech (IT)
 7th Sem.
 Indian Institute of Information Technology Allahabad.
 Ph : 09793899112
 Official Email :: rit2008...@iiita.ac.in
 Another Email :: varunpahwa.ii...@gmail.com

 People who fail to plan are those who plan to fail.

  --
 You received this message because you are subscribed to the Google Groups
 Algorithm Geeks group.
 To post to this group, send email to algogeeks@googlegroups.com.
 To unsubscribe from this group, send email to
 algogeeks+unsubscr...@googlegroups.com.
 For more options, visit this group at
 http://groups.google.com/group/algogeeks?hl=en.




 --
 **Regards
 SAGAR PAREEK
 COMPUTER SCIENCE AND ENGINEERING
 NIT ALLAHABAD




-- 
**Regards
SAGAR PAREEK
COMPUTER SCIENCE AND ENGINEERING
NIT ALLAHABAD

-- 
You received this message because you are subscribed to the Google Groups 
Algorithm Geeks group.
To post to this group, send email to algogeeks@googlegroups.com.
To unsubscribe from this group, send email to 
algogeeks+unsubscr...@googlegroups.com.
For more options, visit this group at 
http://groups.google.com/group/algogeeks?hl=en.



[algogeeks] Puzzle and solution

2011-07-19 Thread sagar pareek
Question :-



Once upon a time in ancient times there was a king who was very fond of
wines.  He had a huge cellar, which had 1000 different varieties of wine all
in different caskets (1000 caskets in all).  In the adjoining kingdom there
was a queen who was envious of the king’s huge wine collection.  After some
time when she could not bear it any more she conspired to kill her by
poisoning all his wine caskets.  So she one sentry to poison all the
caskets, but no sooner had the sentry poisoned only one wine casket that he
was caught and killed by the Royal guards.  Now the king had a major problem
in his hand so as to identify the right casket, which he gave to the
Minister.  Now the position had two peculiar qualities



 Anyone who takes even one drop of poison will die.

 But, he will die only after one month.



 The king also gave the Minister 10 prisoners who could be used as
tasters, cause there lives was of no consequence to the king of kingdom for
that matter, and the Minister was given one month to find the poisoned
casket.  Is it possible for the Minister to find out in one month?  If so
how? If not then how many months are required?





My solution :-

This can be done in one month

Think the solution in binary

ok first i wanna ask u a question :- how many bits are needed to represent
the number 1000 ?
yeah u r right - 10 bits

so here is the solution
let if any prisoner alive it mean it doesnt die and it will be represented
by 1 else if he dies then he will be represented by 0
number the prisoners from 0-9 with 0 the right most (LSB)
now what will be binary representation of 0 ? 00
so if 0th bottle is poisoned then all prisoners must die so taste the
0th(actually 1st) wine to all the prisoners.
what is binary representation of 1? 01
so taste the 1st(actually 2nd) wine to all except the 0th prisoner.
for 2nd, all except 1st (considering 0th as lowest bit) one

and so on.
so at the end if suppose 6th and 2nd prisoner(consider 0 min and 9 max) left
alive then answer will be :- 1*2^5+1*2^1 +1  (note:- here ^= power)


if anyone have more general solution pls let me know

*I hope this is useful  :) :)*



-- 
**Regards
SAGAR PAREEK
COMPUTER SCIENCE AND ENGINEERING
NIT ALLAHABAD

-- 
You received this message because you are subscribed to the Google Groups 
Algorithm Geeks group.
To post to this group, send email to algogeeks@googlegroups.com.
To unsubscribe from this group, send email to 
algogeeks+unsubscr...@googlegroups.com.
For more options, visit this group at 
http://groups.google.com/group/algogeeks?hl=en.



[algogeeks] Re: INFINITY

2011-07-19 Thread SAMMM
Dude I am using solaris with G++ version 2.95.3 . it is giving me
error .. It is not compiled as a C file . it was a cpp file . It
thrown as error write at the screen .

On Jul 19, 6:03 pm, sagar pareek sagarpar...@gmail.com wrote:
 its give me warning
  warning: universal character names are only valid in C++ and C99
 but running...   :)





 On Tue, Jul 19, 2011 at 6:15 PM, SAMMM somnath.nit...@gmail.com wrote:
  It's strange I tried it in GCC version 2.95.3 there is gives error ,
  but in upper version it gives the correct answer .
  I will thinking it will be same for different version .
  I also gone for that , but i was having the intution that it doesn't
  give the right answer .

  Version is a serious mess.

  Moral of the story :--  Keep ur system upgraded .

  On Jul 19, 5:24 pm, varun pahwa varunpahwa2...@gmail.com wrote:
   printf(\u221E\n);

   On Tue, Jul 19, 2011 at 4:10 PM, SAMMM somnath.nit...@gmail.com wrote:
Yaa guys it hav something to do with UNICODE ... I know that 

On Jul 19, 3:32 pm, sagar pareek sagarpar...@gmail.com wrote:
 yeah in my ubuntu too its not printing  :)

 On Tue, Jul 19, 2011 at 3:20 PM, SAMMM somnath.nit...@gmail.com
  wrote:
  But if we use gcc or g++ . In that case it doesn't print it .. Wht
  abt
  tht ...

  On Jul 19, 2:40 pm, Piyush Sinha ecstasy.piy...@gmail.com wrote:
   In Dev C it does

   On Tue, Jul 19, 2011 at 3:08 PM, SAMMM somnath.nit...@gmail.com

wrote:
It doesn't display the infinity symbol.

On Jul 19, 2:24 pm, Piyush Sinha ecstasy.piy...@gmail.com
  wrote:
 *printf(%c\n,236);*

 On Tue, Jul 19, 2011 at 2:45 PM, SAMMM 
  somnath.nit...@gmail.com

  wrote:
  Print the symbol ∞  (INFINITY) through code . Unicode ..

  --
  You received this message because you are subscribed to the
Google
Groups
  Algorithm Geeks group.
  To post to this group, send email to
algogeeks@googlegroups.com.
  To unsubscribe from this group, send email to
  algogeeks+unsubscr...@googlegroups.com.
  For more options, visit this group at
 http://groups.google.com/group/algogeeks?hl=en.

 --
 *Piyush Sinha*
 *IIIT, Allahabad*
 *+91-7483122727*
 * https://www.facebook.com/profile.php?id=10655377926
NEVER
  SAY
 NEVER
 *

--
You received this message because you are subscribed to the
  Google
  Groups
Algorithm Geeks group.
To post to this group, send email to
  algogeeks@googlegroups.com.
To unsubscribe from this group, send email to
algogeeks+unsubscr...@googlegroups.com.
For more options, visit this group at
   http://groups.google.com/group/algogeeks?hl=en.

   --
   *Piyush Sinha*
   *IIIT, Allahabad*
   *+91-7483122727*
   * https://www.facebook.com/profile.php?id=10655377926
  NEVER
SAY
   NEVER
   *- Hide quoted text -

   - Show quoted text -

  --
  You received this message because you are subscribed to the Google
Groups
  Algorithm Geeks group.
  To post to this group, send email to algogeeks@googlegroups.com.
  To unsubscribe from this group, send email to
  algogeeks+unsubscr...@googlegroups.com.
  For more options, visit this group at
 http://groups.google.com/group/algogeeks?hl=en.

 --
 **Regards
 SAGAR PAREEK
 COMPUTER SCIENCE AND ENGINEERING
 NIT ALLAHABAD- Hide quoted text -

 - Show quoted text -

--
You received this message because you are subscribed to the Google
  Groups
Algorithm Geeks group.
To post to this group, send email to algogeeks@googlegroups.com.
To unsubscribe from this group, send email to
algogeeks+unsubscr...@googlegroups.com.
For more options, visit this group at
   http://groups.google.com/group/algogeeks?hl=en.

   --
   Varun Pahwa
   B.Tech (IT)
   7th Sem.
   Indian Institute of Information Technology Allahabad.
   Ph : 09793899112
   Official Email :: rit2008...@iiita.ac.in
   Another Email :: varunpahwa.ii...@gmail.com

   People who fail to plan are those who plan to fail.- Hide quoted text -

   - Show quoted text -

  --
  You received this message because you are subscribed to the Google Groups
  Algorithm Geeks group.
  To post to this group, send email to algogeeks@googlegroups.com.
  To unsubscribe from this group, send email to
  algogeeks+unsubscr...@googlegroups.com.
  For more options, visit this group at
 http://groups.google.com/group/algogeeks?hl=en.

 --
 **Regards
 SAGAR PAREEK
 COMPUTER SCIENCE AND ENGINEERING
 NIT ALLAHABAD- Hide quoted text -

 - Show quoted text -

-- 
You received this message because you are subscribed to the Google Groups 
Algorithm Geeks group.
To post to this group, send email to algogeeks@googlegroups.com.
To unsubscribe from this group, send email to 

Re: [algogeeks] Re: is it possible to detect the first repeating number in a 2-D array (n X n) in O(n) time ?

2011-07-19 Thread snehi jain
@Dumanshu:
i know you have given a good explanation but i have never done
parallel computing..
could you illustrate a bit more especially about the n+1th process..

On Tue, Jul 19, 2011 at 3:35 PM, Bhanu Kishore bhanukishor...@gmail.comwrote:

 @Venkat. That algorithm doesnt work actually.Try for 9,8,1. At 1 , it
 becomes 0.

  --
 You received this message because you are subscribed to the Google Groups
 Algorithm Geeks group.
 To post to this group, send email to algogeeks@googlegroups.com.
 To unsubscribe from this group, send email to
 algogeeks+unsubscr...@googlegroups.com.
 For more options, visit this group at
 http://groups.google.com/group/algogeeks?hl=en.


-- 
You received this message because you are subscribed to the Google Groups 
Algorithm Geeks group.
To post to this group, send email to algogeeks@googlegroups.com.
To unsubscribe from this group, send email to 
algogeeks+unsubscr...@googlegroups.com.
For more options, visit this group at 
http://groups.google.com/group/algogeeks?hl=en.



[algogeeks] Re: INFINITY

2011-07-19 Thread schrodinger
# include stdio.h
int main(){printf(\u221E);}

Compiler details:
gcc (Ubuntu/Linaro 4.5.2-8ubuntu4) 4.5.2

-- 
You received this message because you are subscribed to the Google Groups 
Algorithm Geeks group.
To view this discussion on the web visit 
https://groups.google.com/d/msg/algogeeks/-/hrz9Qheq_XEJ.
To post to this group, send email to algogeeks@googlegroups.com.
To unsubscribe from this group, send email to 
algogeeks+unsubscr...@googlegroups.com.
For more options, visit this group at 
http://groups.google.com/group/algogeeks?hl=en.



Re: [algogeeks] Re: is it possible to detect the first repeating number in a 2-D array (n X n) in O(n) time ?

2011-07-19 Thread ~*~VICKY~*~
Ya sorry abt that my algo is wrong!

On Tue, Jul 19, 2011 at 3:35 PM, Bhanu Kishore bhanukishor...@gmail.comwrote:

 @Venkat. That algorithm doesnt work actually.Try for 9,8,1. At 1 , it
 becomes 0.

  --
 You received this message because you are subscribed to the Google Groups
 Algorithm Geeks group.
 To post to this group, send email to algogeeks@googlegroups.com.
 To unsubscribe from this group, send email to
 algogeeks+unsubscr...@googlegroups.com.
 For more options, visit this group at
 http://groups.google.com/group/algogeeks?hl=en.




-- 
Cheers,

  Vicky

-- 
You received this message because you are subscribed to the Google Groups 
Algorithm Geeks group.
To post to this group, send email to algogeeks@googlegroups.com.
To unsubscribe from this group, send email to 
algogeeks+unsubscr...@googlegroups.com.
For more options, visit this group at 
http://groups.google.com/group/algogeeks?hl=en.



[algogeeks] how to optimally compute a matrix (nXn) to the power of k?

2011-07-19 Thread snehi jain
question had come in Amazon  interview ...

-- 
You received this message because you are subscribed to the Google Groups 
Algorithm Geeks group.
To post to this group, send email to algogeeks@googlegroups.com.
To unsubscribe from this group, send email to 
algogeeks+unsubscr...@googlegroups.com.
For more options, visit this group at 
http://groups.google.com/group/algogeeks?hl=en.



[algogeeks] Re: how to optimally compute a matrix (nXn) to the power of k?

2011-07-19 Thread SAMMM

This is done by using exponential theory :-


 I am giving my code here for a^b


int power (int a,int b)
{
   int x=1,y=a;
   while(b0)
   {
   if(b%2==1)  x=(x*y)
   if (b/=2) y=(y*y);
   }
 return x;
}


Run time  O(log b)

-- 
You received this message because you are subscribed to the Google Groups 
Algorithm Geeks group.
To post to this group, send email to algogeeks@googlegroups.com.
To unsubscribe from this group, send email to 
algogeeks+unsubscr...@googlegroups.com.
For more options, visit this group at 
http://groups.google.com/group/algogeeks?hl=en.



Re: [algogeeks] Re: c++ smart pointers

2011-07-19 Thread Deepthi Srinivasan
Implementation of smart pointer:

template class T class *auto_ptr*
{
T* ptr;public:
explicit *auto_ptr*(T* p = 0) : ptr(p) {}
*~auto_ptr*() {delete ptr;}
T *operator**()  {return *ptr;}
T* *operator-*() {return ptr;}
// ...};

Its all there in this link:

http://ootips.org/yonat/4dev/smart-pointers.html


On Mon, Jul 18, 2011 at 7:45 PM, DK divyekap...@gmail.com wrote:

 I would suggest that you open the include files of your C++ compiler
 distribution and see how the auto_ptr class is implemented.
 It's not difficult.

 Here's how it's implemented in GCC.
 (see attachment).

 Just ignore all the GCC rubbish and the uglified names.

 --
 DK

 http://twitter.com/divyekapoor
 http://www.divye.in

  --
 You received this message because you are subscribed to the Google Groups
 Algorithm Geeks group.
 To view this discussion on the web visit
 https://groups.google.com/d/msg/algogeeks/-/BaL5bORA0EwJ.

 To post to this group, send email to algogeeks@googlegroups.com.
 To unsubscribe from this group, send email to
 algogeeks+unsubscr...@googlegroups.com.
 For more options, visit this group at
 http://groups.google.com/group/algogeeks?hl=en.


-- 
You received this message because you are subscribed to the Google Groups 
Algorithm Geeks group.
To post to this group, send email to algogeeks@googlegroups.com.
To unsubscribe from this group, send email to 
algogeeks+unsubscr...@googlegroups.com.
For more options, visit this group at 
http://groups.google.com/group/algogeeks?hl=en.



Re: [algogeeks] Re: how this output is obtained ?

2011-07-19 Thread DeVaNsH gUpTa
@Vengadanathan:
I think you should see this link
http://c-faq.com/expr/seqpoints.html
-- 
Thanks and Regards
*Devansh Gupta*
*B.Tech Third Year*
*MNNIT, Allahabad*

-- 
You received this message because you are subscribed to the Google Groups 
Algorithm Geeks group.
To post to this group, send email to algogeeks@googlegroups.com.
To unsubscribe from this group, send email to 
algogeeks+unsubscr...@googlegroups.com.
For more options, visit this group at 
http://groups.google.com/group/algogeeks?hl=en.



Re: [algogeeks] Re: EXPLAIN THE OUTPUTS

2011-07-19 Thread geek forgeek
@schrodinger  y a[] value is not lost in first call.it should be lost in
first call only?

On Tue, Jul 19, 2011 at 8:24 PM, schrodinger 6fae1ce6347...@gmail.comwrote:

 First output of memory location is fine.
 Second output is also expected one.
 Third output will vary compiler to compiler and from time to time.

 This is because a[] is a local  to fun(). First time when you call
 printf(%d\n,r[0]) its fine. but after executing printf() location of r is
 lost and hence you'll get different output is undefined.

  --
 You received this message because you are subscribed to the Google Groups
 Algorithm Geeks group.
 To view this discussion on the web visit
 https://groups.google.com/d/msg/algogeeks/-/gzaXX7gzytAJ.

 To post to this group, send email to algogeeks@googlegroups.com.
 To unsubscribe from this group, send email to
 algogeeks+unsubscr...@googlegroups.com.
 For more options, visit this group at
 http://groups.google.com/group/algogeeks?hl=en.


-- 
You received this message because you are subscribed to the Google Groups 
Algorithm Geeks group.
To post to this group, send email to algogeeks@googlegroups.com.
To unsubscribe from this group, send email to 
algogeeks+unsubscr...@googlegroups.com.
For more options, visit this group at 
http://groups.google.com/group/algogeeks?hl=en.



[algogeeks] Re: Puzzle and solution

2011-07-19 Thread sagar pareek
hey guys pls tell any other better solution ...

On Tue, Jul 19, 2011 at 6:41 PM, sagar pareek sagarpar...@gmail.com wrote:

 Question :-



 Once upon a time in ancient times there was a king who was very fond of
 wines.  He had a huge cellar, which had 1000 different varieties of wine
 all in different caskets (1000 caskets in all).  In the adjoining kingdom
 there was a queen who was envious of the king’s huge wine collection.  After
 some time when she could not bear it any more she conspired to kill her by
 poisoning all his wine caskets.  So she one sentry to poison all the
 caskets, but no sooner had the sentry poisoned only one wine casket that he
 was caught and killed by the Royal guards.  Now the king had a major
 problem in his hand so as to identify the right casket, which he gave to the
 Minister.  Now the position had two peculiar qualities



  Anyone who takes even one drop of poison will die.

  But, he will die only after one month.



  The king also gave the Minister 10 prisoners who could be used as
 tasters, cause there lives was of no consequence to the king of kingdom for
 that matter, and the Minister was given one month to find the poisoned
 casket.  Is it possible for the Minister to find out in one month?  If so
 how? If not then how many months are required?





 My solution :-

 This can be done in one month

 Think the solution in binary

 ok first i wanna ask u a question :- how many bits are needed to represent
 the number 1000 ?
 yeah u r right - 10 bits

 so here is the solution
 let if any prisoner alive it mean it doesnt die and it will be represented
 by 1 else if he dies then he will be represented by 0
 number the prisoners from 0-9 with 0 the right most (LSB)
 now what will be binary representation of 0 ? 00
 so if 0th bottle is poisoned then all prisoners must die so taste the
 0th(actually 1st) wine to all the prisoners.
 what is binary representation of 1? 01
 so taste the 1st(actually 2nd) wine to all except the 0th prisoner.
 for 2nd, all except 1st (considering 0th as lowest bit) one

 and so on.
 so at the end if suppose 6th and 2nd prisoner(consider 0 min and 9 max)
 left alive then answer will be :- 1*2^5+1*2^1 +1  (note:- here ^= power)


 if anyone have more general solution pls let me know

 *I hope this is useful  :) :)*



 --
 **Regards
 SAGAR PAREEK
 COMPUTER SCIENCE AND ENGINEERING
 NIT ALLAHABAD




-- 
**Regards
SAGAR PAREEK
COMPUTER SCIENCE AND ENGINEERING
NIT ALLAHABAD

-- 
You received this message because you are subscribed to the Google Groups 
Algorithm Geeks group.
To post to this group, send email to algogeeks@googlegroups.com.
To unsubscribe from this group, send email to 
algogeeks+unsubscr...@googlegroups.com.
For more options, visit this group at 
http://groups.google.com/group/algogeeks?hl=en.



Re: [algogeeks] Re: c++ smart pointers

2011-07-19 Thread Saurabh
Thanks all  :)

On Tue, Jul 19, 2011 at 8:37 PM, Deepthi Srinivasan deeps1...@gmail.comwrote:

 Implementation of smart pointer:

 template class T class *auto_ptr*
 {
 T* ptr;public:
 explicit *auto_ptr*(T* p = 0) : ptr(p) {}
 *~auto_ptr*() {delete ptr;}
 T *operator**()  {return *ptr;}
 T* *operator-*() {return ptr;}
 // ...};

 Its all there in this link:

 http://ootips.org/yonat/4dev/smart-pointers.html


 On Mon, Jul 18, 2011 at 7:45 PM, DK divyekap...@gmail.com wrote:

 I would suggest that you open the include files of your C++ compiler
 distribution and see how the auto_ptr class is implemented.
 It's not difficult.

 Here's how it's implemented in GCC.
 (see attachment).

 Just ignore all the GCC rubbish and the uglified names.

 --
 DK

 http://twitter.com/divyekapoor
 http://www.divye.in

  --
 You received this message because you are subscribed to the Google Groups
 Algorithm Geeks group.
 To view this discussion on the web visit
 https://groups.google.com/d/msg/algogeeks/-/BaL5bORA0EwJ.

 To post to this group, send email to algogeeks@googlegroups.com.
 To unsubscribe from this group, send email to
 algogeeks+unsubscr...@googlegroups.com.
 For more options, visit this group at
 http://groups.google.com/group/algogeeks?hl=en.


  --
 You received this message because you are subscribed to the Google Groups
 Algorithm Geeks group.
 To post to this group, send email to algogeeks@googlegroups.com.
 To unsubscribe from this group, send email to
 algogeeks+unsubscr...@googlegroups.com.
 For more options, visit this group at
 http://groups.google.com/group/algogeeks?hl=en.




-- 
Regards  n Luv
Saurabh Badhai

-- 
You received this message because you are subscribed to the Google Groups 
Algorithm Geeks group.
To post to this group, send email to algogeeks@googlegroups.com.
To unsubscribe from this group, send email to 
algogeeks+unsubscr...@googlegroups.com.
For more options, visit this group at 
http://groups.google.com/group/algogeeks?hl=en.



[algogeeks] interview questoin

2011-07-19 Thread pacific :-)
Find unique string from a list of strings in one pass.

-- 
regards,
chinna.

-- 
You received this message because you are subscribed to the Google Groups 
Algorithm Geeks group.
To post to this group, send email to algogeeks@googlegroups.com.
To unsubscribe from this group, send email to 
algogeeks+unsubscr...@googlegroups.com.
For more options, visit this group at 
http://groups.google.com/group/algogeeks?hl=en.



Re: [algogeeks] Re: interview questoin

2011-07-19 Thread pacific :-)
sorry.

On Tue, Jul 19, 2011 at 9:07 PM, Shubham Maheshwari 
shubham.veloc...@gmail.com wrote:

 what is meant by unique string ...!!

 A string which occurs only once.


 On Tue, Jul 19, 2011 at 9:04 PM, SAMMM somnath.nit...@gmail.com wrote:

 There is only one unique string  in the list of strings (words) ?

 There can be many.



 On Jul 19, 8:31 pm, pacific :-) pacific4...@gmail.com wrote:
  Find unique string from a list of strings in one pass.
 
  --
  regards,
  chinna.

 --
 You received this message because you are subscribed to the Google Groups
 Algorithm Geeks group.
 To post to this group, send email to algogeeks@googlegroups.com.
 To unsubscribe from this group, send email to
 algogeeks+unsubscr...@googlegroups.com.
 For more options, visit this group at
 http://groups.google.com/group/algogeeks?hl=en.




 --
 Shubham Maheshwari
 ShubZz
 O.o o.O

 enJoY ...!!!

  --
 You received this message because you are subscribed to the Google Groups
 Algorithm Geeks group.
 To post to this group, send email to algogeeks@googlegroups.com.
 To unsubscribe from this group, send email to
 algogeeks+unsubscr...@googlegroups.com.
 For more options, visit this group at
 http://groups.google.com/group/algogeeks?hl=en.




-- 
regards,
chinna.

-- 
You received this message because you are subscribed to the Google Groups 
Algorithm Geeks group.
To post to this group, send email to algogeeks@googlegroups.com.
To unsubscribe from this group, send email to 
algogeeks+unsubscr...@googlegroups.com.
For more options, visit this group at 
http://groups.google.com/group/algogeeks?hl=en.



[algogeeks] Circle Circle more Circles .........

2011-07-19 Thread SAMMM
Suppose N number of circle is given with their x  y coordinate and
radius . Now the question is to find the total area covered by the N
circles .. Circles can be overlapping depending on their coordinates .

-- 
You received this message because you are subscribed to the Google Groups 
Algorithm Geeks group.
To post to this group, send email to algogeeks@googlegroups.com.
To unsubscribe from this group, send email to 
algogeeks+unsubscr...@googlegroups.com.
For more options, visit this group at 
http://groups.google.com/group/algogeeks?hl=en.



Re: [algogeeks] Circle Circle more Circles .........

2011-07-19 Thread priyanka goel
can u pl tell wat is dis x  y coordinate?
are dey centre coordinates or any point on circumference of circle..

-- 
You received this message because you are subscribed to the Google Groups 
Algorithm Geeks group.
To post to this group, send email to algogeeks@googlegroups.com.
To unsubscribe from this group, send email to 
algogeeks+unsubscr...@googlegroups.com.
For more options, visit this group at 
http://groups.google.com/group/algogeeks?hl=en.



Re: [algogeeks] Re: how to optimally compute a matrix (nXn) to the power of k?

2011-07-19 Thread Rishabh Maurya
You can try like this

#include stdio.h
#include string.h
#define N 3
void copy(int A[N][N],int B[N][N])
{
int i,j;
for(i=0;iN;i++)
for(j=0;jN;j++)
A[i][j]=B[i][j];
}
void mul(int A[N][N],int B[N][N],int M[N][N])
{
int i,j,k;
for(i=0;iN;i++)
for(j=0;jN;j++) {
M[i][j]=0;
for(k=0;kN;k++)
M[i][j]+=A[i][k]*B[k][j];
}
}
void exp(int M[N][N],int n)
{
int R[N][N];
int A[N][N];
int B[N][N];
int i;
memset(R,0,sizeof(R));
for(i=0;iN;i++)
R[i][i]=1;
while(n0) {
if(n%2==1) {
copy(A,R);
mul(M,A,R);
}
n=n1;
copy(A,M);
copy(B,M);
mul(A,B,M);
}
copy(M,R);
}
int main()
{
int i,j;
int M[N][N];
int n;
for(i=0;iN;i++)
for(j=0;jN;j++)
scanf(%d,M[i][j]);
scanf(%d,n);
exp(M,n);
for(i=0;iN;i++)
for(j=0;jN;j++)
printf(%d%c,M[i][j],j==N-1?'\n':' ');
return 0;
}

-- 
You received this message because you are subscribed to the Google Groups 
Algorithm Geeks group.
To post to this group, send email to algogeeks@googlegroups.com.
To unsubscribe from this group, send email to 
algogeeks+unsubscr...@googlegroups.com.
For more options, visit this group at 
http://groups.google.com/group/algogeeks?hl=en.



Re: [algogeeks] Re: how to optimally compute a matrix (nXn) to the power of k?

2011-07-19 Thread Rishabh Maurya
time complexity is (cost of multiplication)*log(n)

-- 
You received this message because you are subscribed to the Google Groups 
Algorithm Geeks group.
To post to this group, send email to algogeeks@googlegroups.com.
To unsubscribe from this group, send email to 
algogeeks+unsubscr...@googlegroups.com.
For more options, visit this group at 
http://groups.google.com/group/algogeeks?hl=en.



Re: [algogeeks] Re: how to optimally compute a matrix (nXn) to the power of k?

2011-07-19 Thread priyanka goel
is this code running?

-- 
You received this message because you are subscribed to the Google Groups 
Algorithm Geeks group.
To post to this group, send email to algogeeks@googlegroups.com.
To unsubscribe from this group, send email to 
algogeeks+unsubscr...@googlegroups.com.
For more options, visit this group at 
http://groups.google.com/group/algogeeks?hl=en.



[algogeeks] interview question

2011-07-19 Thread nidhi jain
Consider a c++ template funtion
templateclass T
T Add(T a, T b)
{return a+b ;}
if this function is called as T c = Add(SAM, SUNG); what will
happen? What is the problem in the template declaration/ How to solve
the problem.

-- 
You received this message because you are subscribed to the Google Groups 
Algorithm Geeks group.
To post to this group, send email to algogeeks@googlegroups.com.
To unsubscribe from this group, send email to 
algogeeks+unsubscr...@googlegroups.com.
For more options, visit this group at 
http://groups.google.com/group/algogeeks?hl=en.



Re: [algogeeks] Re: how to optimally compute a matrix (nXn) to the power of k?

2011-07-19 Thread Rishabh Maurya
Yes its running fine at gcc 4.3.2  . And it might show warning in that case
just change the name of the function exp().

-- 
You received this message because you are subscribed to the Google Groups 
Algorithm Geeks group.
To post to this group, send email to algogeeks@googlegroups.com.
To unsubscribe from this group, send email to 
algogeeks+unsubscr...@googlegroups.com.
For more options, visit this group at 
http://groups.google.com/group/algogeeks?hl=en.



[algogeeks] Re: Circle Circle more Circles .........

2011-07-19 Thread SAMMM
See the input will be :-

6 No of circles

x1 y1 R1
x2 y2 R2
x3 y3 R3
x4 y4 R4
x5 y5 R5
x6 y6 R6

Output:-
Area occupied by the above circles (one line) 4 decimal points .


On Jul 19, 9:01 pm, priyanka goel priya888g...@gmail.com wrote:
 can u pl tell wat is dis x  y coordinate?
 are dey centre coordinates or any point on circumference of circle..

-- 
You received this message because you are subscribed to the Google Groups 
Algorithm Geeks group.
To post to this group, send email to algogeeks@googlegroups.com.
To unsubscribe from this group, send email to 
algogeeks+unsubscr...@googlegroups.com.
For more options, visit this group at 
http://groups.google.com/group/algogeeks?hl=en.



[algogeeks] MS

2011-07-19 Thread swetha rahul
Hi,
Find the kth smallest element in O(logk) given 2 sorted arrays.
Merging the arrays is not allowed. I can do it in O(k).. How to do in
O(logk)..

-- 
You received this message because you are subscribed to the Google Groups 
Algorithm Geeks group.
To post to this group, send email to algogeeks@googlegroups.com.
To unsubscribe from this group, send email to 
algogeeks+unsubscr...@googlegroups.com.
For more options, visit this group at 
http://groups.google.com/group/algogeeks?hl=en.



Re: [algogeeks] Re: how to optimally compute a matrix (nXn) to the power of k?

2011-07-19 Thread snehi jain
@Rishab  Thanks ..
could you explain what your are doing in the function exp()..


On Tue, Jul 19, 2011 at 9:47 PM, Rishabh Maurya poofiefoo...@gmail.comwrote:

 Yes its running fine at gcc 4.3.2  . And it might show warning in that case
 just change the name of the function exp().

  --
 You received this message because you are subscribed to the Google Groups
 Algorithm Geeks group.
 To post to this group, send email to algogeeks@googlegroups.com.
 To unsubscribe from this group, send email to
 algogeeks+unsubscr...@googlegroups.com.
 For more options, visit this group at
 http://groups.google.com/group/algogeeks?hl=en.


-- 
You received this message because you are subscribed to the Google Groups 
Algorithm Geeks group.
To post to this group, send email to algogeeks@googlegroups.com.
To unsubscribe from this group, send email to 
algogeeks+unsubscr...@googlegroups.com.
For more options, visit this group at 
http://groups.google.com/group/algogeeks?hl=en.



Re: [algogeeks] interview question

2011-07-19 Thread Anika Jain
here T becomes char *.. u r trying to add two addreses here...

On Tue, Jul 19, 2011 at 9:46 PM, nidhi jain nidhi.jain311...@gmail.comwrote:

 Consider a c++ template funtion
 templateclass T
 T Add(T a, T b)
 {return a+b ;}
 if this function is called as T c = Add(SAM, SUNG); what will
 happen? What is the problem in the template declaration/ How to solve
 the problem.

 --
 You received this message because you are subscribed to the Google Groups
 Algorithm Geeks group.
 To post to this group, send email to algogeeks@googlegroups.com.
 To unsubscribe from this group, send email to
 algogeeks+unsubscr...@googlegroups.com.
 For more options, visit this group at
 http://groups.google.com/group/algogeeks?hl=en.



-- 
You received this message because you are subscribed to the Google Groups 
Algorithm Geeks group.
To post to this group, send email to algogeeks@googlegroups.com.
To unsubscribe from this group, send email to 
algogeeks+unsubscr...@googlegroups.com.
For more options, visit this group at 
http://groups.google.com/group/algogeeks?hl=en.



Re: [algogeeks] Re: how to optimally compute a matrix (nXn) to the power of k?

2011-07-19 Thread LALIT SHARMA
@rishabh  , thnX a lot :P

Plz explain further  :P

On Tue, Jul 19, 2011 at 10:15 PM, snehi jain snehijai...@gmail.com wrote:

 @Rishab  Thanks ..
 could you explain what your are doing in the function exp()..


 On Tue, Jul 19, 2011 at 9:47 PM, Rishabh Maurya poofiefoo...@gmail.comwrote:

 Yes its running fine at gcc 4.3.2  . And it might show warning in that
 case just change the name of the function exp().

  --
 You received this message because you are subscribed to the Google Groups
 Algorithm Geeks group.
 To post to this group, send email to algogeeks@googlegroups.com.
 To unsubscribe from this group, send email to
 algogeeks+unsubscr...@googlegroups.com.
 For more options, visit this group at
 http://groups.google.com/group/algogeeks?hl=en.


  --
 You received this message because you are subscribed to the Google Groups
 Algorithm Geeks group.
 To post to this group, send email to algogeeks@googlegroups.com.
 To unsubscribe from this group, send email to
 algogeeks+unsubscr...@googlegroups.com.
 For more options, visit this group at
 http://groups.google.com/group/algogeeks?hl=en.




-- 
Lalit Kishore Sharma,

IIIT Allahabad,
7th Sem.
Contact No. - +919670057056 , +918957935169

-- 
You received this message because you are subscribed to the Google Groups 
Algorithm Geeks group.
To post to this group, send email to algogeeks@googlegroups.com.
To unsubscribe from this group, send email to 
algogeeks+unsubscr...@googlegroups.com.
For more options, visit this group at 
http://groups.google.com/group/algogeeks?hl=en.



[algogeeks] Re: how to optimally compute a matrix (nXn) to the power of k?

2011-07-19 Thread SAMMM
The logic which I have posted , it works on that principle only . Take
a good look . It is the standard way fopr finding the power of a
number . (a^b)

You take and example for a=3 and b=4 and test it in My posted code .
then you will understand wht is happening .



On Jul 19, 9:45 pm, snehi jain snehijai...@gmail.com wrote:
 @Rishab  Thanks ..
 could you explain what your are doing in the function exp()..

 On Tue, Jul 19, 2011 at 9:47 PM, Rishabh Maurya poofiefoo...@gmail.comwrote:



  Yes its running fine at gcc 4.3.2  . And it might show warning in that case
  just change the name of the function exp().

   --
  You received this message because you are subscribed to the Google Groups
  Algorithm Geeks group.
  To post to this group, send email to algogeeks@googlegroups.com.
  To unsubscribe from this group, send email to
  algogeeks+unsubscr...@googlegroups.com.
  For more options, visit this group at
 http://groups.google.com/group/algogeeks?hl=en.- Hide quoted text -

 - Show quoted text -

-- 
You received this message because you are subscribed to the Google Groups 
Algorithm Geeks group.
To post to this group, send email to algogeeks@googlegroups.com.
To unsubscribe from this group, send email to 
algogeeks+unsubscr...@googlegroups.com.
For more options, visit this group at 
http://groups.google.com/group/algogeeks?hl=en.



[algogeeks] Microsoft Interview Qn - Looping

2011-07-19 Thread Reynald
Given the following program, MS will be printed, infinite number of
times:
   int n = 20;
   int i;
   for (i=0; in; i--)
   printf(MS);

 Apply one of the following three operations one at a time such that
MS can be printed 20 times.
1. You can add only one character into it.
2. you can delete only one character in it.
3. You can replace a character by another character.

 Find all possible solutions for this problem.

-- 
You received this message because you are subscribed to the Google Groups 
Algorithm Geeks group.
To post to this group, send email to algogeeks@googlegroups.com.
To unsubscribe from this group, send email to 
algogeeks+unsubscr...@googlegroups.com.
For more options, visit this group at 
http://groups.google.com/group/algogeeks?hl=en.



Re: [algogeeks] Re: how to optimally compute a matrix (nXn) to the power of k?

2011-07-19 Thread Rishabh Maurya
Yes sure ,

we know if A B C are matrices of same dimension then Ax(BxC)=(AxB)xC

So now what SAMM is saying i.e normal exponential of any number can be
applied in case of matrix expo too and the same thing I too did but using
loop without recursive fashion.

I hope its clear now .

-- 
You received this message because you are subscribed to the Google Groups 
Algorithm Geeks group.
To post to this group, send email to algogeeks@googlegroups.com.
To unsubscribe from this group, send email to 
algogeeks+unsubscr...@googlegroups.com.
For more options, visit this group at 
http://groups.google.com/group/algogeeks?hl=en.



[algogeeks] Re: how to optimally compute a matrix (nXn) to the power of k?

2011-07-19 Thread SAMMM
Let me explain a little more :-

For example :-


a^b = Can be written as (a^2)^(b/2)  For n is even .  [ Reason:-  For
b/2  in my code orn=n1;  in Rishabh code ]
  = Can be written as a.(a^2)^(b-1/2)  For n is odd [ Reason :-
x=(x*y)  for the odd number for getting the alone  ]


It works on this principle . Give it a try . In my code You just
have to substitute 'a' By a matrix . That will server the purpose ..


I hope you doubth wil be clear .. As said earlier work with the
numbers first then you will get the logic .


On Jul 19, 9:54 pm, SAMMM somnath.nit...@gmail.com wrote:
 The logic which I have posted , it works on that principle only . Take
 a good look . It is the standard way fopr finding the power of a
 number . (a^b)

 You take and example for a=3 and b=4 and test it in My posted code .
 then you will understand wht is happening .

 On Jul 19, 9:45 pm, snehi jain snehijai...@gmail.com wrote:



  @Rishab  Thanks ..
  could you explain what your are doing in the function exp()..

  On Tue, Jul 19, 2011 at 9:47 PM, Rishabh Maurya 
  poofiefoo...@gmail.comwrote:

   Yes its running fine at gcc 4.3.2  . And it might show warning in that 
   case
   just change the name of the function exp().

    --
   You received this message because you are subscribed to the Google Groups
   Algorithm Geeks group.
   To post to this group, send email to algogeeks@googlegroups.com.
   To unsubscribe from this group, send email to
   algogeeks+unsubscr...@googlegroups.com.
   For more options, visit this group at
  http://groups.google.com/group/algogeeks?hl=en.-Hide quoted text -

  - Show quoted text -- Hide quoted text -

 - Show quoted text -

-- 
You received this message because you are subscribed to the Google Groups 
Algorithm Geeks group.
To post to this group, send email to algogeeks@googlegroups.com.
To unsubscribe from this group, send email to 
algogeeks+unsubscr...@googlegroups.com.
For more options, visit this group at 
http://groups.google.com/group/algogeeks?hl=en.



Re: [algogeeks] Microsoft Interview Qn - Looping

2011-07-19 Thread Radhika Renganathan
i know 3 solns!

1)  int n = 20;
  int i;
  for (i=0; i+n; i--)
  printf(MS);
2)  int n = 20;
  int i;
  for (i=0; in; n--)
  printf(MS);

3) int n = 20;
  int i;
  for (i=0; -in; i--)
  printf(MS);

On Tue, Jul 19, 2011 at 10:27 PM, Reynald reynaldsus...@gmail.com wrote:

 Given the following program, MS will be printed, infinite number of
 times:
   int n = 20;
   int i;
   for (i=0; in; i--)
   printf(MS);

  Apply one of the following three operations one at a time such that
MS can be printed 20 times.
1. You can add only one character into it.
2. you can delete only one character in it.
3. You can replace a character by another character.

  Find all possible solutions for this problem.

 --
 You received this message because you are subscribed to the Google Groups
 Algorithm Geeks group.
 To post to this group, send email to algogeeks@googlegroups.com.
 To unsubscribe from this group, send email to
 algogeeks+unsubscr...@googlegroups.com.
 For more options, visit this group at
 http://groups.google.com/group/algogeeks?hl=en.




-- 
 radhika .. :)

-- 
You received this message because you are subscribed to the Google Groups 
Algorithm Geeks group.
To post to this group, send email to algogeeks@googlegroups.com.
To unsubscribe from this group, send email to 
algogeeks+unsubscr...@googlegroups.com.
For more options, visit this group at 
http://groups.google.com/group/algogeeks?hl=en.



Re: [algogeeks] MS

2011-07-19 Thread Piyush Sinha
are the sizes of the two arrays same??

On 7/19/11, swetha rahul swetharahu...@gmail.com wrote:
 Hi,
 Find the kth smallest element in O(logk) given 2 sorted arrays.
 Merging the arrays is not allowed. I can do it in O(k).. How to do in
 O(logk)..

 --
 You received this message because you are subscribed to the Google Groups
 Algorithm Geeks group.
 To post to this group, send email to algogeeks@googlegroups.com.
 To unsubscribe from this group, send email to
 algogeeks+unsubscr...@googlegroups.com.
 For more options, visit this group at
 http://groups.google.com/group/algogeeks?hl=en.




-- 
*Piyush Sinha*
*IIIT, Allahabad*
*+91-7483122727*
* https://www.facebook.com/profile.php?id=10655377926 NEVER SAY
NEVER
*

-- 
You received this message because you are subscribed to the Google Groups 
Algorithm Geeks group.
To post to this group, send email to algogeeks@googlegroups.com.
To unsubscribe from this group, send email to 
algogeeks+unsubscr...@googlegroups.com.
For more options, visit this group at 
http://groups.google.com/group/algogeeks?hl=en.



Re: [algogeeks] MS

2011-07-19 Thread swetha rahul
Arrays are not of the same size

On Tue, Jul 19, 2011 at 10:41 PM, Rishabh Maurya poofiefoo...@gmail.comwrote:

 Its solvable using Binary Search , offcourse  not in log(k) but in log(sum
 of size of array).

 --
 You received this message because you are subscribed to the Google Groups
 Algorithm Geeks group.
 To post to this group, send email to algogeeks@googlegroups.com.
 To unsubscribe from this group, send email to
 algogeeks+unsubscr...@googlegroups.com.
 For more options, visit this group at
 http://groups.google.com/group/algogeeks?hl=en.


-- 
You received this message because you are subscribed to the Google Groups 
Algorithm Geeks group.
To post to this group, send email to algogeeks@googlegroups.com.
To unsubscribe from this group, send email to 
algogeeks+unsubscr...@googlegroups.com.
For more options, visit this group at 
http://groups.google.com/group/algogeeks?hl=en.



[algogeeks] MS ques

2011-07-19 Thread siva viknesh
Given an infinite stream of bits with bits being appended at the
highest significant position. Give an algorithm to say whether the
number formed by sequence of bits that had been processed till then ,
is divisible by 3 or not ?


My sol:

have a variable sum...find the sum of bitswhenever u add a bit
do sum+=bit value  ... check whether sum%3==0.
Is my solution ok?? anyother good solutions ??

-- 
You received this message because you are subscribed to the Google Groups 
Algorithm Geeks group.
To post to this group, send email to algogeeks@googlegroups.com.
To unsubscribe from this group, send email to 
algogeeks+unsubscr...@googlegroups.com.
For more options, visit this group at 
http://groups.google.com/group/algogeeks?hl=en.



Re: [algogeeks] MS ques

2011-07-19 Thread sudhanshu pandey
use automata theory. draw dfa for divisibility by 3..

On Tue, Jul 19, 2011 at 11:23 PM, siva viknesh sivavikne...@gmail.comwrote:

 Given an infinite stream of bits with bits being appended at the
 highest significant position. Give an algorithm to say whether the
 number formed by sequence of bits that had been processed till then ,
 is divisible by 3 or not ?


 My sol:

 have a variable sum...find the sum of bitswhenever u add a bit
 do sum+=bit value  ... check whether sum%3==0.
 Is my solution ok?? anyother good solutions ??

 --
 You received this message because you are subscribed to the Google Groups
 Algorithm Geeks group.
 To post to this group, send email to algogeeks@googlegroups.com.
 To unsubscribe from this group, send email to
 algogeeks+unsubscr...@googlegroups.com.
 For more options, visit this group at
 http://groups.google.com/group/algogeeks?hl=en.




-- 
SUDHANSHU PANDEY

--only fair thing in this world is a chance--

-- 
You received this message because you are subscribed to the Google Groups 
Algorithm Geeks group.
To post to this group, send email to algogeeks@googlegroups.com.
To unsubscribe from this group, send email to 
algogeeks+unsubscr...@googlegroups.com.
For more options, visit this group at 
http://groups.google.com/group/algogeeks?hl=en.



  1   2   >